Health Assessment Exam 4

¡Supera tus tareas y exámenes ahora con Quizwiz!

c

Abused women have been found to have significantly more health problems, including a. cardiovascular disease. b. chronic anemia. c. chronic pain. d. cancer.

Lateral nodes

Along the humerus, inside the upper arm. From the central axillary node's drains flow up to the intraclavicular and supraclavicular nodes

Pectoral nodes (anterior)

Along the lateral edge of the pectoralis major muscle, just inside the anterior axillary fold

Subscapular nodes (posterior)

Along the lateral edge of the scapula, deep in the posterior axillary fold

Fixation of the breast:

Hey symmetry, distortion, or decreased mobility with the elevated are maneuver. I cancer becomes invasive, the fibrous sticks the breast to the underlying pectoral muscle. Here note the right breast is held against the chest wall.

Central axillary nodes

High up in the middle of the axilla, over the ribs and serratas anterior muscle. These receive length from the other three groups of nodes

The nurse is conducting a heritage assessment. Which question is most appropriate for this assessment? a. What is your religion? b. Do you mostly participate in the religious traditions of your family? c. Do you smoke? d. Do you have a history of heart disease?

B

Male breast cancer

1% of her and men. There's no standard screening mammogram. Presents as a painless palpable mass— hard, irregular, nontender, fix to the area. May also have nipple retraction. Nipple discharge, with or without a palpable mass is a significant morning of early breast cancer. often diagnosed later due to lack of screening and general awareness mean age is 60 to 70 years of age

The nurse is discussing breast self-examination with a postmenopausal woman. The best time for postmenopausal women to perform breast self-examination is: a. the same day every month. b. daily, during the shower or bath. c. 1 week after her menstrual period. d. every year with her annual gynecologic examination.

A Postmenopausal women are no longer experiencing regular menstrual cycles but need to continue to perform breast self-examination on a monthly basis. Choosing the same day of the month is a helpful reminder to perform breast self-examination.

. A 30-year-old woman has recently moved to the United States with her husband. They are living with the womans sister until they can get a home of their own. When company arrives to visit with the womans sister, the woman feels suddenly shy and retreats to the back bedroom to hide until the company leaves. She explains that her reaction to guests is simply because she does not know how to speak perfect English. This woman could be experiencing: a. Culture shock. b. Cultural taboos. c. Cultural unfamiliarity. d. Culture disorientation.

A

. A woman who has lived in the United States for a year after moving from Europe has learned to speak English and is almost finished with her college studies. She now dresses like her peers and says that her family in Europe would hardly recognize her. This nurse recognizes that this situation illustrates which concept? a. Assimilation b. Heritage consistency c. Biculturalism d. Acculturation

A

An Asian-American woman is experiencing diarrhea, which is believed to be cold or yin. The nurse expects that the woman is likely to try to treat it with: a. Foods that are hot or yang. b. Readings and Eastern medicine meditations. c. High doses of medicines believed to be cold. d. No treatment is tried because diarrhea is an expected part of life.

A

During an assessment, the nurse notices that a patient is handling a small charm that is tied to a leather strip around his neck. Which action by the nurse is appropriate? a. Ask the patient about the item and its significance. b. Ask the patient to lock the item with other valuables in the hospitals safe. c. Tell the patient that a family member should take valuables home. d. No action is necessary.

A

Illness is considered part of lifes rhythmic course and is an outward sign of disharmony within. This statement most accurately reflects the views about illness from which theory? a. Naturalistic b. Biomedical c. Reductionist d. Magicoreligious

A

The nurse recognizes that working with children with a different cultural perspective may be especially difficult because: a. Children have spiritual needs that are influenced by their stages of development. b. Children have spiritual needs that are direct reflections of what is occurring in their homes. c. Religious beliefs rarely affect the parents perceptions of the illness. d. Parents are often the decision makers, and they have no knowledge of their childrens spiritual needs.

A

When reviewing the demographics of ethnic groups in the United States, the nurse recalls that the largest and fastest growing population is: a. Hispanic. b. Black. c. Asian. d. American Indian.

A

A 43-year-old woman is at the clinic for a routine examination. She reports that she has had a breast lump in her right breast for years. Recently, it has begun to change in consistency and is becoming harder. She reports that 5 years ago her physician evaluated the lump and determined that it "was nothing to worry about." *The examination validates the presence of a mass in the right upper outer quadrant at 1 o'clock, approximately 5 cm from the nipple. It is firm, mobile, nontender, with borders that are not well defined. The nurse's recommendation to her is:* a. "Because of the change in consistency of the lump, it should be further evaluated by a physician." b. "The changes could be related to your menstrual cycles. Keep track of changes in the mass each month." c. "This is probably nothing to worry about because it has been present for years and was determined to be noncancerous at that time." d. "Because you are experiencing no pain and the size has not changed, continue to monitor the lump and return to the clinic in 3 months."

A A lump that has been present for years and is not exhibiting changes may not be serious but still should be explored. Any recent change or new lump should be evaluated. The other responses are not correct.

During an examination, the nurse notes a supernumerary nipple just under the patient's left breast. The patient tells the nurse that she always thought it was a mole. Which statement about this finding is correct? a. It is a normal variation and not a significant finding. b. It is a significant finding and needs further investigation. c. It also contains glandular tissue and may leak milk during pregnancy and lactation. d. The patient is correct-it is actually a mole that happens to be located under the breast.

A A supernumerary nipple looks like a mole, but close examination reveals a tiny nipple and areola. It is not a significant finding.

A 14-year-old girl is anxious about not having reached menarche. When taking the history, the nurse should ascertain which of the following? The age: a. she began to develop breasts b. her mother developed breasts c. she began to develop pubic hair d. she began to develop axillary hair.

A Full development from stage 2 to stage 5 takes an average of 3 years, although the range is 1.5 to 6 years. Pubic hair develops during this time, and axillary hair appears 2 years after the onset of pubic hair. The beginning of breast development precedes menarche by about 2 years. Menarche occurs in breast development stage 3 or 4, usually just after the peak of the adolescent growth spurt, which occurs around age 12 years. See Figure 17-6.

A woman has just learned that she is pregnant. What are some things the nurse should teach her about her breasts? a. She can expect her areolae to become larger and darker in color. b. Breasts may begin secreting milk after the fourth month of pregnancy. c. She should inspect her breasts for visible veins and report this immediately. d. During pregnancy, breast changes are fairly uncommon; most of the changes occur after the birth.

A The areolae become larger and grow a darker brown as pregnancy progresses, and the tubercles become more prominent. (The brown color fades after lactation, but the areolae never return to the original color). A venous pattern is prominent over the skin surface and does not need to be reported as it is an expected finding. After the fourth month, colostrum, a thick, yellow fluid (precursor to milk) may be expressed from the breasts.

The nurse is reviewing statistics regarding breast cancer. Which woman, aged 40 years in the United States, has the highest risk for development of breast cancer? a. African-American b. White c. Asian d. American Indian

A The incidence of breast cancer varies with different cultural groups. White women have a higher incidence of breast cancer than African-American women starting at age 45 years; but African-American women have a higher incidence before age 45 years. Asian, Hispanic, and American Indian women have a lower risk for development of breast cancer (American Cancer Society, 2009-2010).

The nurse is palpating a female patient's breasts during an examination. Which of these positions is most likely to make significant lumps more distinct during breast palpation? a. Supine with arms raised over her head b. Sitting with arms relaxed at the sides c. Supine with arms relaxed at the sides d. Sitting with arms flexed and fingertips touching shoulders

A The nurse should help the woman to a supine position, tuck a small pad under the side to be palpated, and help the woman raise her arm over her head. These maneuvers will flatten the breast tissue and displace it medially. Any significant lumps will then feel more distinct.

A new mother calls the clinic to report that part of her left breast is red, swollen, tender, very hot, and hard. She has a fever of 101 degrees F. She has also had symptoms of the flu, such as chills, sweating, and feeling tired. The nurse notices that she has been breastfeeding for 1 month. From her description, what condition does the nurse suspect? a. Mastitis b. Paget's disease c. Plugged milk duct d. Mammary duct ectasia

A The symptoms describe mastitis, which stems from infection or stasis caused by a plugged duct. A plugged duct does not have infection present. (See Table 17-7.) Refer to Table 17-6 for descriptions of Paget's disease and mammary duct ectasia.

3. During a speculum inspection of the vagina, the nurse would expect to see what at the end of the vaginal canal? a. Cervix b. Uterus c. Ovaries d. Fallopian tubes

A At the end of the canal, the uterine cervix projects into the vagina. DIF: Cognitive Level: Remembering (Knowledge) REF: p. 738 MSC: Client Needs: Physiologic Integrity: Physiologic Adaptation

36. When assessing a newborn infant's genitalia, the nurse notices that the genitalia are somewhat engorged. The labia majora are swollen, the clitoris looks large, and the hymen is thick. The vaginal opening is difficult to visualize. The infant's mother states that she is worried about the labia being swollen. The nurse should reply: a. "This is a normal finding in newborns and should resolve within a few weeks." b. "This finding could indicate an abnormality and may need to be evaluated by a physician." c. "We will need to have estrogen levels evaluated to ensure that they are within normal limits." d. "We will need to keep close watch over the next few days to see if the genitalia decrease in size."

A It is normal for a newborn's genitalia to be somewhat engorged. A sanguineous vaginal discharge or leukorrhea is normal during the first few weeks because of the maternal estrogen effect. During the early weeks, the genital engorgement resolves, and the labia minora atrophy and remain small until puberty. DIF: Cognitive Level: Applying (Application) REF: p. 759 MSC: Client Needs: Health Promotion and Maintenance

31. During an examination, which tests will the nurse collect to screen for cervical cancer? a. Endocervical specimen, cervical scrape, and vaginal pool b. Endocervical specimen, vaginal pool, and acetic acid wash c. Endocervical specimen, potassium hydroxide (KOH) preparation, and acetic acid wash d. Cervical scrape, acetic acid wash, saline mount ("wet prep")

A Laboratories may vary in method, but usually the test consists of three specimens: endocervical specimen, cervical scrape, and vaginal pool. The other tests (acetic acid wash, KOH preparation, and saline mount) are used to test for sexually transmitted infections. DIF: Cognitive Level: Understanding (Comprehension) REF: pp. 752-753 MSC: Client Needs: Safe and Effective Care Environment: Management of Care

10. A woman is in the clinic for an annual gynecologic examination. The nurse should plan to begin the interview with the: a. Menstrual history, because it is generally nonthreatening. b. Obstetric history, because it includes the most important information. c. Urinary system history, because problems may develop in this area as well. d. Sexual history, because discussing it first will build rapport.

A Menstrual history is usually nonthreatening and therefore a good topic with which to begin the interview. Obstetric, urinary, and sexual histories are also part of the interview but not necessarily the best topics with which to start. DIF: Cognitive Level: Applying (Application) REF: p. 740 MSC: Client Needs: Safe and Effective Care Environment: Management of Care

23. The nurse is preparing to interview a postmenopausal woman. Which of these statements is true as it applies to obtaining the health history of a postmenopausal woman? a. The nurse should ask a postmenopausal woman if she has ever had vaginal bleeding. b. Once a woman reaches menopause, the nurse does not need to ask any history questions. c. The nurse should screen for monthly breast tenderness. d. Postmenopausal women are not at risk for contracting STIs; therefore, these questions can be omitted.

A Postmenopausal bleeding warrants further workup and referral. The other statements are not true. DIF: Cognitive Level: Understanding (Comprehension) REF: p. 744 MSC: Client Needs: Physiologic Integrity: Reduction of Risk Potential

39. When performing an external genitalia examination of a 10-year-old girl, the nurse notices that no pubic hair has grown in and the mons and the labia are covered with fine vellus hair. These findings are consistent with stage _____ of sexual maturity, according to the Sexual Maturity Rating scale. a. 1 b. 2 c. 3 d. 4

A Sexual Maturity Rating stage 1 is the preadolescent stage. There is no pubic hair, and the mons and labia are covered with fine, vellus hair as on the abdomen (see Table 26-1). DIF: Cognitive Level: Applying (Application) REF: p. 739 MSC: Client Needs: Health Promotion and Maintenance

2. During an examination, the nurse observes a female patient's vestibule and expects to see the: a. Urethral meatus and vaginal orifice. b. Vaginal orifice and vestibular (Bartholin) glands. c. Urethral meatus and paraurethral (Skene) glands. d. Paraurethral (Skene) and vestibular (Bartholin) glands.

A The labial structures encircle a boat-shaped space, or cleft, termed the vestibule. Within the vestibule are numerous openings. The urethral meatus and vaginal orifice are visible. The ducts of the paraurethral (Skene) glands and the vestibular (Bartholin) glands are present but not visible. DIF: Cognitive Level: Understanding (Comprehension) REF: p. 737 MSC: Client Needs: Physiologic Integrity: Physiologic Adaptation

4. The uterus is usually positioned tilting forward and superior to the bladder. This position is known as: a. Anteverted and anteflexed. b. Retroverted and anteflexed. c. Retroverted and retroflexed. d. Superiorverted and anteflexed.

A The uterus is freely movable, not fixed, and usually tilts forward and superior to the bladder (a position labeled as anteverted and anteflexed). DIF: Cognitive Level: Remembering (Knowledge) REF: p. 738 MSC: Client Needs: General

8. The nurse is reviewing the changes that occur with menopause. Which changes are associated with menopause? a. Uterine and ovarian atrophy, along with a thinning of the vaginal epithelium b. Ovarian atrophy, increased vaginal secretions, and increasing clitoral size c. Cervical hypertrophy, ovarian atrophy, and increased acidity of vaginal secretions d. Vaginal mucosa fragility, increased acidity of vaginal secretions, and uterine hypertrophy

A The uterus shrinks because of its decreased myometrium. The ovaries atrophy to 1 to 2 cm and are not palpable after menopause. The sacral ligaments relax, and the pelvic musculature weakens; consequently, the uterus droops. The cervix shrinks and looks paler with a thick glistening epithelium. The vaginal epithelium atrophies, becoming thinner, drier, and itchy. The vaginal pH becomes more alkaline, and secretions are decreased, which results in a fragile mucosal surface that is at risk for vaginitis. DIF: Cognitive Level: Understanding (Comprehension) REF: p. 740 MSC: Client Needs: Health Promotion and Maintenance

37. During a vaginal examination of a 38-year-old woman, the nurse notices that the vulva and vagina are erythematous and edematous with thick, white, curdlike discharge adhering to the vaginal walls. The woman reports intense pruritus and thick white discharge from her vagina. The nurse knows that these history and physical examination findings are most consistent with which condition? a. Candidiasis b. Trichomoniasis c. Atrophic vaginitis d. Bacterial vaginosis

A The woman with candidiasis often reports intense pruritus and thick white discharge. The vulva and vagina are erythematous and edematous. The discharge is usually thick, white, and curdlike. Infection with trichomoniasis causes a profuse, watery, gray-green, and frothy discharge. Bacterial vaginosis causes a profuse discharge that has a "foul, fishy, rotten" odor. Atrophic vaginitis may have a mucoid discharge. (See Table 26-5 for complete descriptions of each option.) DIF: Cognitive Level: Analyzing (Analysis) REF: p. 768 MSC: Client Needs: Physiologic Integrity: Physiologic Adaptation

Intraductal papilloma

A discrete benign tumor that arises in a single or multiple papillary duct. May have serious or zero sanguinous discharge. Most common in women as age 40 to 60. Most are benign, but multiple at high-risk of subsequent cancers.

Edema (Peau d'Orange) in the breast

A lymphatic obstruction producing edema. Thickening the skin and exaggerating the hair follicles, giving a pig skin or orange peel like look. This condition suggest cancer.

a

A woman seeks medical attention for a cut made by a knife during a physical assault. The health care provider would document the cut as an a. incision b. ecchymosis. c. avulsion. d. abrasion.

The nurse is examining a 62-year-old man and notes that he has gynecomastia bilaterally. The nurse should explore his history for which related conditions? *Select all that apply.* a. Obesity b. Malnutrition c. Hyperthyroidism d. Type 2 diabetes mellitus e. Liver disease f. History of alcohol abuse

A, C, E, F Gynecomastia occurs with obesity, Cushing's syndrome, liver cirrhosis, adrenal disease, hyperthyroidism, and numerous drugs: alcohol and marijuana use, estrogen treatment for prostate cancer, antibiotics (metronidazole, isoniazid), digoxin, ACE inhibitors, diazepam, and tricyclic antidepressants.

The nurse is assessing the breasts of a 68-year-old woman and discovers a mass in the upper outer quadrant of the left breast. When assessing this mass, the nurse keeps in mind that characteristics of a cancerous mass include which of the following? *Select all that apply.* a. Nontender mass b. Dull, heavy pain on palpation c. Rubbery texture and mobile d. Hard, dense, and immobile e. Regular border f. Irregular, poorly delineated border

A, D, F Cancerous breast masses are solitary, unilateral, nontender, masses. They are solid, hard, dense, and fixed to underlying tissues or skin as cancer becomes invasive. Their borders are irregular and poorly delineated. They are often painless, although the person may have pain. They are most common in upper outer quadrant. A dull, heavy pain on palpation and a mass with a rubbery texture and a regular border are characteristics of benign breast disease.

The nurse is performing a nutritional assessment on an 80-year-old patient. The nurse knows that physiologic changes that directly affect the nutritional status of the elderly include: 1.slowed gastrointestinal motility. 2.hyperstimulation of the salivary glands. 3.an increased sensitivity to spicy and aromatic foods. 4.decreased gastrointestinal absorption causing esophageal reflux.

ANS: 1 Normal physiological changes in aging adults that affect nutritional status include slowed gastrointestinal motility, decreased gastrointestinal absorption, diminished olfactory and taste sensitivity, decreased saliva production, decreased visual acuity, and poor dentition.

If a 29-year-old woman weighs 146 pounds and the nurse determines her ideal body weight to be 120 pounds, how would the nurse classify the woman's weight? 1.Obese 2.Mildly overweight 3.Suffering from malnutrition 4.Within appropriate range of ideal weight

ANS: 1 Obesity is defined as greater than 120% of ideal body weight. For this patient, 120% of her ideal body weight, 120 pounds, is 144 pounds. Her current weight of 146 pounds is greater than 120% of ideal body weight.

During a nutritional assessment, why is it important for the nurse to ask a patient what medications he or she is taking? 1.Certain drugs can affect the metabolism of nutrients. 2.The nurse needs to assess the patient for allergic reactions. 3.Medications need to be documented on the record for the physician's review. 4.Medications can affect one's memory and ability to identify food eaten in the last 24 hours.

ANS: 1 Analgesics, antacids, anticonvulsants, antibiotics, diuretics, laxatives, antineoplastic drugs, steroids, and oral contraceptives are among the drugs that can interact with nutrients, impairing their digestion, absorption, metabolism, or use.

The nurse is performing a nutritional assessment on a 15-year-old girl, who tells the nurse that she is "so fat." Assessment reveals that she is 5 feet 4 inches and weighs 110 pounds. The nurse's appropriate response would be: 1."How much do you think you should weigh?" 2."Don't worry about it, you're not that overweight." 3."The best thing for you would be to go on a diet." 4."I used to always think I was fat when I was your age."

ANS: 1 Because of adolescents' increased body awareness and self-consciousness, they are prone to develop eating disorders such as anorexia nervosa or bulimia, conditions in which the real or perceived body image does not compare favorably to an ideal image.

The nurse is providing nutrition information to the mother of a 1-year-old child. Which of the following statements represents accurate information for this age group? 1.It is important to maintain adequate fat and caloric intake. 2.The recommended dietary allowances for an infant are the same as for an adolescent. 3.At this age the baby's growth is minimal so caloric requirements are decreased. 4.The baby should be placed on skim milk to decrease the risk of coronary artery disease when older.

ANS: 1 Because of rapid growth, especially of the brain, infants and children younger than 2 years should not drink skim or low-fat milk or be placed on low-fat diets—fat (calories and essential fatty acids) is required for proper growth and central nervous system development.

In teaching a patient how to determine total body fat at home, the nurse includes instructions to obtain measurements of: 1.height and weight. 2.frame size and weight. 3.waist and hip circumferences. 4.mid upper arm circumference and arm span.

ANS: 1 Body mass index, calculated by using height and weight measurements, is a practical marker of optimal weight for height and an indicator of obesity.

The nurse is providing care for a 68-year-old woman who is complaining of constipation. What concern exists regarding her nutritional status? 1.The absorption of nutrients may be impaired. 2.The constipation may represent a food allergy. 3.She may need emergency surgery for the problem. 4.The gastrointestinal problem will increase her caloric demand.

ANS: 1 Gastrointestinal symptoms such as vomiting, diarrhea, or constipation may interfere with nutrient intake or absorption.

After completing a diet assessment on a 30-year-old woman, the nurse suspects that she may be deficient in iron. Laboratory studies to obtain to verify this condition would be: 1.hemoglobin and hematocrit. 2.cholesterol and triglycerides. 3.creatinine and serum protein. 4.serum albumin and urinary urea nitrogen.

ANS: 1 The hemoglobin determination is used to detect iron-deficiency anemia. Hematocrit, a measure of cell volume, is also an indicator of iron status.

When considering a nutritional assessment, the nurse is aware that the most common anthropometric measurements include: 1.height and weight. 2.leg circumference. 3.biceps skinfold thickness. 4.hip and waist measurement.

ANS: 1 The most commonly used anthropometric measures are height, weight, triceps skinfold thickness, elbow breadth, and arm and head circumferences.

The nurse is evaluating patients for obesity-related diseases. Which one of the following would be at increased risk? 1.A 29-year-old woman whose percent ideal body weight is 125%, waist is 33 inches, and hips 36 inches 2.A 32-year-old man whose percent ideal body weight is 115%, waist is 34 inches, and hips 36 inches 3.A 38-year-old man whose percent ideal body weight is 120%, waist is 35 inches, and hips 38 inches 4.A 46-year old woman whose percent ideal body weight is 130%, waist is 30 inches, and hips 38 inches

ANS: 1 The waist-to-hip ratio assesses body fat distribution as an indicator of health risk. A waist-to-hip ratio of 1.0 or greater in men or 0.8 or greater in women is indicative of android (upper body obesity) and increasing risk for obesity-related disease and early death.

The nurse recognizes that which of the following persons is at greatest risk for undernutrition? 1.A 5-month-old infant 2.A 50-year-old woman 3.A 20-year-old college student 4.A 30-year-old hospital administrator

ANS: 1 Vulnerable groups are infants, children, pregnant women, recent immigrants, persons with low incomes, hospitalized people, and aging adults.

During an assessment of a patient who has been homeless for several years, the nurse notices that his tongue is magenta in color. This is an indication of: 1.iron deficiency. 2.riboflavin deficiency. 3.vitamin D and calcium deficiency. 4.vitamin C deficiency.

ANS: 2 "Magenta tongue" is a sign of riboflavin deficiency. In contrast, a pale tongue is attributable to iron deficiency. Vitamin D and calcium deficiency cause osteomalacia in adults, and vitamin C deficiency causes scorbutic gums; a beefy red-colored tongue is caused by vitamin-B complex deficiency.

A mother and her 13-year-old daughter express their concern related to the daughter's recent weight gain and increase in appetite. Which of the following represents information the nurse should discuss with them? 1.The necessity of exercise and dieting at this age 2.Suggestions for snacks high in protein, iron, and calcium 3.Teenagers who have a weight problem should not be allowed to snack 4.The importance of a low-calorie diet to prevent the accumulation of fat

ANS: 2 After a period of slow growth in late childhood, adolescence is characterized by rapid physical growth and endocrine and hormonal changes. Caloric and protein requirements increase to meet this demand. Because of bone growth and increasing muscle mass (and, in girls, the onset of menarche), calcium and iron requirements also increase.

During a nutritional assessment of a 22-year-old male refugee, the nurse must remember to: 1.obtain a 24-hour dietary recall. 2.clarify what is meant by the term "food." 3.provide him with a standard dietary handbook. 4.assume that his diet is consistent with other refugees from the same country.

ANS: 2 Although one may assume that the term "food" is a universal concept, the person should clarify what is meant by the term.

When evaluating the results of laboratory tests, the nurse knows that which of the following statements is true? 1.Normal values do not vary according to age. 2.Variations based on biocultural differences may exist. 3.It is not necessary to repeat laboratory tests once malnutrition has been determined. 4.Lab tests are more sensitive than other parts of the assessment and should take priority.

ANS: 2 Biocultural variations occur with some laboratory tests, such as hemoglobin/ hematocrit, serum cholesterol, and serum transferrin.

The nurse needs to determine the body mass index of an 80-year-old man who is confined to a wheelchair. Which of the following is true in this situation? 1.Changes in fat distribution will affect the waist-to-hip ratio. 2.Height measurements may not be accurate because of changes in bone. 3.Declining muscle mass will affect the triceps skinfold measure. 4.Mid arm circumference is difficult to obtain because of loss of skin elasticity.

ANS: 2 Height measures may not be accurate in individuals confined to a bed or wheelchair or those over 60 years of age (because of osteoporotic changes).

In performing an assessment on a 49-year-old woman who has imbalanced nutrition as a result of dysphagia, which of the following data would the nurse expect to find? 1.An increase in hair growth 2.Inadequate food intake 3.Weight 10% to 20% over ideal 4.Sore, inflamed buccal cavity

ANS: 2 Imbalanced nutrition, less than body requirements, would result from inadequate food intake.

A 21-year-old woman has been on a low-protein liquid diet for the past 2 months. She has had adequate calories and appears well nourished. In further assessing her, what would the nurse expect to find? 1.Poor skin turgor 2.Decreased serum albumin 3.Increased lymphocyte count 4.Triceps skinfold less than standard

ANS: 2 Kwashiorkor (protein malnutrition) is due to diets that may be high in calories but contain little or no protein (e.g., low-protein liquid diets, fad diets, and long-term use of dextrose-containing intravenous fluids). Serum albumin <3.5 g/dl.

Which of the following factors is most likely to affect the nutritional status of an 82-year-old person? 1.Increase in taste and smell 2.Living alone on a fixed income 3.Change in cardiovascular status 4.Increase in gastrointestinal motility and absorption

ANS: 2 Socioeconomic conditions frequently have the greatest effect on the nutritional status of the aging adult; these factors should be closely evaluated. Physical limitations, income, and social isolation are frequent problems and can obviously interfere with the acquisition of a balanced diet.

Which of the following statements is true concerning the nutritional assessment? 1.It is only useful in patients who are overweight. 2.It identifies patients who are at risk of malnutrition. 3.This assessment can only be thoroughly done by a dietician. 4.It provides the nurse with physical findings related to all the systems.

ANS: 2 The purposes of the nutritional assessment are to: (1) identify individuals who are malnourished or are at risk for development of malnutrition, (2) provide data for designing a plan of care that will prevent or minimize the development of malnutrition, and (3) establish baseline data for evaluating the efficacy of nutritional care.

A patient tells the nurse that his food just doesn't have any taste anymore. The nurse's best response would be: 1."That must be really frustrating." 2."When did you first notice this change?" 3."My food doesn't always have a lot of taste either." 4."Sometimes that happens but your taste will come back."

ANS: 2 With changes in appetite, taste, smell, or chewing or swallowing, the examiner asks about the type of change and when the change occurred.

The nurse is assessing an obese patient for signs of metabolic syndrome. This condition is diagnosed when three or more certain risk factors are present. Which of the following are risk factors for metabolic syndrome? Select all that apply. 1.Fasting plasma glucose level less than 110 mg/dl 2.Fasting plasma glucose level greater than or equal to 100 mg/dl 3.Blood pressure reading of 140/90 mm/Hg 4.Blood pressure reading of 110/80 mm/Hg 5.Triglyceride level of 120 mg/dl

ANS: 2, 3 Metabolic syndrome is diagnosed when 3 out of 5 of the following risk factors are present: (1) fasting plasma glucose level ≥100 mg/dl or DT for elevated glucose; (2) blood pressure ≥130 systolic or 85 diastolic mm Hg or DT for hypertension; (3) waist circumference ≥40 inches for men and ≥35 inches for women; (4) high-density lipoprotein cholesterol <40 mg/dL in men and <50 mg/dL in women or DT for reduced HDL-C; and (5) triglyceride levels ≥150 mg/dL or DT for elevated triglycerides.

A pregnant woman who is human immunodeficiency virus (HIV) positive is asking the nurse about breast-feeding her baby. Which of the following statement is true? 1.There is not enough information to know whether it would be safe for her to breast-feed. 2.It is safe for women who are HIV positive to breast-feed. 3.Women who are HIV positive should not breast-feed because HIV can be transmitted through breast milk. 4.She can breastfeed as long as she uses pumped breast milk.

ANS: 3 Although relatively few contraindications to breast-feeding exist, women who are HIV positive should not breast-feed because HIV can be transmitted through breast milk.

The mother of an 8-year-old boy is concerned about the amount of weight her son has gained. To determine whether this is a problem, the nurse will measure: 1.arm span. 2.waist-to-hip ratio. 3.skinfold thickness. 4.mid upper arm circumference.

ANS: 3 Determination of skinfold thickness or body mass index may be useful in evaluating childhood and teenage overnutrition.

A 16-year-old girl is being seen at the clinic for gastrointestinal complaints and weight loss. The nurse determines that many of her complaints may be related to erratic eating patterns, eating predominantly fast foods, and high caffeine intake. In this situation, which of the following is most appropriate when collecting current dietary intake information? 1.Schedule a time for direct observation of the adolescent during meals. 2.Ask the patient for a 24-hour diet recall and assume this is reflective of a typical day for her. 3.Have the patient complete a food diary for 3 days—2 weekdays and 1 weekend day. 4.Use the food frequency questionnaire to identify the amount of intake of specific foods.

ANS: 3 Food diaries require the individual to write down everything consumed for a certain time period. Because of the erratic eating patterns of this individual, assessing dietary intake over a few days would produce more accurate information regarding eating patterns. Direct observation is best used with young children or older adults.

The nurse is assessing a 30-year-old unemployed immigrant from an underdeveloped country who has been in the United States for 1 month. Which of the following problems related to his nutritional status might the nurse expect to find? 1.Obesity 2.Hypotension 3.Osteomalacia 4.Coronary artery disease

ANS: 3 General undernutrition, hypertension, diarrhea, lactose intolerance, osteomalacia (soft bones), scurvy, and dental caries are among the more common nutrition- related problems of new immigrants from developing countries.

Which of the following interventions is most appropriate when the nurse is planning nutritional interventions for a healthy, active 74-year-old woman? 1.Decrease the amount of carbohydrates to prevent lean muscle catabolism. 2.Increase the amount of soy and tofu in her diet to promote bone growth and reverse osteoporosis. 3.Decrease the number of calories she is eating because of the decrease in energy requirements from loss of lean body mass. 4.Increase the number of calories she is eating because of the increased energy needs of the elderly.

ANS: 3 Important nutritional features of the older years are a decrease in energy require- ments as a result of loss of lean body mass, the most metabolically active tissue, and an increase in fat mass.

42. A 65-year-old man is brought to the emergency department after he was found dazed and incoherent, alone in his apartment. He has an enlarged liver and is moderately dehydrated. When evaluating his serum albumin level, the nurse must keep in mind that: 1.serum albumin levels will increase as liver function decreases. 2.serum albumin levels are a sensitive measure of early protein malnutrition. 3.low serum albumin levels may be caused by reasons other than protein-calorie malnutrition. 4.the results of the serum albumin measurement along with the patient's hemoglobin level should be considered.

ANS: 3 Low serum albumin levels may be caused by reasons other than protein-calorie malnutrition, such as an altered hydration status and decreased liver function.

Which of the following conditions is due to an inadequate intake of both protein and calories? 1.Obesity 2.Bulimia 3.Marasmus 4.Kwashiorkor

ANS: 3 Marasmus, protein-calorie malnutrition, is due to an inadequate intake of protein and calories or prolonged starvation.

The nurse is concerned about the skeletal protein reserves of a patient who has been hospitalized frequently for chronic lung disease. Which of the following measurements would be necessary to include in the assessment? 1.Body mass index 2.Weight and height 3.Mid arm muscle area 4.Ideal body weight and frame size

ANS: 3 Mid arm muscle area is a good indicator of lean body mass and skeletal protein reserves. These reserves are important in growing children and are especially valuable in evaluating persons who may be malnourished because of chronic illness, multiple surgeries, or inadequate dietary intake. The equation for calculating mid arm muscle area includes mid upper arm circumference and mid-upper arm muscle circumference.

To perform a triceps skinfold assessment, the examiner would do which of the following? 1.After pinching the skin and fat, apply the calipers vertically to the fat fold. 2.Gently pinch the skin and fat on the anterior aspect of the patient's arm and then apply calipers. 3.After applying the calipers, wait 3 seconds before taking a reading. Repeat the procedure three times. 4.Instruct the patient to stand with the back to the examiner and arms folded across the chest and pinch the skin on the forearm.

ANS: 3 Release the lever of the calipers while holding the skinfold. Wait 3 seconds and then take a reading. Repeat three times and average the three skinfold measurements.

When the mid upper arm circumference and triceps skinfold of an 82-year- old man are evaluated, it is important to remember that: 1.these measurements are no longer necessary for the elderly. 2.derived weight measures may be difficult to interpret because of wide ranges of normal. 3.these measurements may not be accurate because of changes in skin and fat distribution. 4.measurements may be difficult to obtain if the patient is unable to flex his elbow to at least 90 degrees.

ANS: 3 Remember that accurate mid upper arm circumference and triceps skinfold measurements are difficult to obtain and interpret in older adults because of sagging skin, changes in fat distribution, and declining muscle mass.

To obtain an accurate nitrogen balance and creatinine-height index, the nurse must ensure that: 1.the patient's height and weight have been determined. 2.the laboratory draws the blood specimen in the early morning. 3.an accurate 24-hour urine specimen has been collected. 4.the patient has not had anything by mouth for 8 hours before the test.

ANS: 3 The validities of creatinine-height index and nitrogen balance studies are dependent on the accuracy of the 24-hour urine collection.

Which of the following statements is true regarding routine laboratory testing in the following individuals 1.In pregnancy, no laboratory testing is needed unless problems with the pregnancy are suspected. 2.In the elderly, laboratory values regarding cholesterol and triglycerides are the most important because of the risk of disease. 3.During adolescence, unless disease is suspected, laboratory evaluation of hemoglobin and hematocrit levels and urinalysis for glucose and protein are adequate. 4.In infancy and childhood, laboratory tests should be performed at each well-child check-up, regardless of whether the child is exhibiting signs of illness that affect nutritional status.

ANS: 3 Unless disease is suspected, evaluation of hemoglobin and hematocrit levels and urinalysis for glucose and protein levels are adequate in adolescents. In infancy, laboratory tests are performed only if nutritional problems are suspected or illnesses affect nutritional status. Many laboratory values are monitored during pregnancy and older adulthood.

Which of the following individuals is most likely to have an anergic response when assessing skin test antigens? 1.An 8-year-old child 2.An obese individual 3.A healthy 80-year-old female 4.An individual with malnutrition

ANS: 4 Adequate immunity can be assessed by a positive reaction to multiple skin test antigens. Antigens are injected and the response is noted at 24 and 48 hours. Anergy occurs with malnutrition, hepatic failure, infection, and immunosuppressive drugs.

A pregnant woman is interested in breast-feeding her baby, and asks several questions about the topic. Which information is appropriate for the nurse to share with her? 1.Breast-feeding is best when also supplemented with bottle feedings. 2.Babies who are breastfed often require supplemental vitamins. 3.Breast-feeding is recommended for infants for the first 2 years of life. 4.Breast milk provides the nutrients necessary for growth as well as natural immunity.

ANS: 4 Breast-feeding is recommended for full-term infants for the first year of life because breast milk is ideally formulated to promote normal infant growth and development and natural immunity.

Which body composition-measuring tool measures fat and lean body mass and bone mineral density? 1.Waist-to-hip ratio 2.Body mass index 3.Bioelectrical impedance analysis 4.Dual-energy x-ray (DEXA) absorptiometry

ANS: 4 Dual-energy x-ray (DEXA) absorptiometry measures bone mineral density and fat and lean body mass.

Which of the following best describes the technique for measuring frame size? 1.With the patient standing, measure the distance from the top of the head to the back of the heel. 2.With the patient in a sitting position, measure the distance from the condyle of the humerus to the clavicle. 3.With the patient's right arm extended forward and the elbow extended, measure the distance from fingertips to the condyle of the humerus. 4.With the right arm extended forward and the elbow bent, use the calipers to measure the distance between the condyles of the humerus.

ANS: 4 Instruct the person to extend the right arm forward, perpendicular to the body. Bend the elbow to a 90-degree angle with the palm of the hand turned laterally. Facing the person, place the calipers on the condyles of the humerus. Read the distance between the condyles.

A 40-year-old man has had a recent weight loss of 20 pounds because of a quick loss diet and stress. He tells the nurse that he keeps getting "colds and the flu." In addition to assessing his nutrition status, the nurse would also want to obtain which laboratory report? 1.Serum albumin 2.Cholesterol level 3.Serum transferrin 4.Total lymphocyte count

ANS: 4 Loss of immunocompetence is strongly correlated with malnutrition in stressed and starving patients. The most commonly used tests of immune function are total lymphocyte count and skin testing, also called delayed cutaneous hypersensitivity.

When assessing muscle mass and fat stores on a 40-year-old woman, the nurse would use: 1.triceps skinfold. 2.mid arm muscle area. 3.percent ideal body weight. 4.mid upper arm circumference.

ANS: 4 Mid upper arm circumference estimates skeletal muscle mass and fat stores.

When assessing a patient's nutritional status, the nurse recalls that the best definition of optimal nutritional status is: 1.nutrients in excess of daily body requirements. 2.sufficient nutrients to provide for the minimum body needs. 3.sufficient nutrients for daily body requirements but not for increased metabolic demands. 4.sufficient nutrients to provide for daily body requirements and for increased metabolic demands.

ANS: 4 Optimal nutritional status is achieved when sufficient nutrients are consumed to support day-to-day body needs and any increased metabolic demands resulting from growth, pregnancy, or illness.

The nurse is seeing a patient for the first time who has no history of nutrition-related problems. The initial nutritional screening should include which of the following? 1.Calorie count of nutrients 2.Anthropometric measures 3.Complete physical examination 4.Measurement of weight and weight history

ANS: 4 Parameters used for nutrition screening typically include weight and weight history, conditions associated with increased nutritional risk, diet information, and routine laboratory data.

Which of the following measurements is an early indicator of protein malnutrition? 1.Serum albumin 2.Serum creatinine 3.Nitrogen balance 4.Serum transferrin

ANS: 4 Serum transferrin, with a half-life of 8 to 10 days, may be a more sensitive indicator of visceral protein status than albumin. Serum albumin has a relatively long half-life of 17 to 20 days.

The nurse is discussing appropriate foods with the mother of a 3-year-old child. Which of the following foods are recommended? 1.Foods that the child will eat, no matter what they are 2.Foods easy to hold such as hot dogs, nuts, and grapes 3.Any foods as long as the rest of the family is eating them 4.Finger foods and nutritious snacks that can't cause choking.

ANS: 4 Use of small portions, finger foods, simple meals, and nutritious snacks are strategies to improve dietary intake. Foods likely to be aspirated should be avoided (e.g., hot dogs, nuts, grapes, round candies, popcorn).

A patient is asked to indicate on a form how many times he eats a specific food. This would describe which of the following methods for obtaining dietary information? 1.Food diary 2.Calorie count 3.24-hour recall 4.Food frequency questionnaire

ANS: 4 With this tool, information is collected on how many times per day, week, or month the individual eats particular foods.

10. The nurse is assessing bruising on an injured patient. Which color indicates a new bruise that is less than 2 hours old? A. Red B. Purple-blue C. Greenish-brown D. Brownish-yellow

ANS: A A new bruise is usually red and will often develop a purple or purple-blue appearance 12 to 36 hours after blunt-force trauma. The color of bruises (and ecchymoses) generally progresses from purple-blue to bluish-green to greenish-brown to brownish-yellow before fading away.

5. An 11-year-old girl is in the clinic for a sports physical examination. The nurse notices that she has begun to develop breasts, and during the conversation the girl reveals that she is worried about her development. The nurse should use which of these techniques to best assist the young girl in understanding the expected sequence for development? The nurse should: a. Use the Tanner scale on the five stages of sexual development. b. Describe her development and compare it with that of other girls her age. c. Use the Jacobsen table on expected development on the basis of height and weight data. d. Reassure her that her development is within normal limits and tell her not to worry about the next step.

ANS: A The Tanner scale on the five stages of pubic hair development is helpful in teaching girls the expected sequence of sexual development (see Table 26-1). The other responses are not appropriate. DIF: Cognitive Level: Applying (Application) REF: p. 739 MSC: Client Needs: Health Promotion and Maintenance

7. When documenting intimate partner violence and elder abuse, the nurse should include: A. Photographic documentation of injuries. B. A summary of the abused patient's statements. C. Verbatim documentation of every statement made. D. A general description of injuries in the progress notes.

ANS: A Documentation of intimate partner violence and elder abuse must include detailed nonbiased progress notes, the use of injury maps, and photographic documentation. Written documentation needs to be verbatim, within reason. Not every statement can be documented.

The mother of a 5-year-old girl tells the nurse that she has noticed her daughter "scratching at her bottom a lot the last few days." During the assessment, the nurse finds redness and raised skin in the anal area. This finding most likely indicates: a. Pinworms. b. Chickenpox. c. Constipation. d. Bacterial infection.

ANS: A In children, pinworms are a common cause of intense itching and irritated anal skin. The other options are not correct. DIF: Cognitive Level: Analyzing (Analysis) REF: p. 725 MSC: Client Needs: Physiologic Integrity: Physiologic Adaptation

A 59-year-old patient has been diagnosed with prostatitis and is being seen at the clinic for complaints of burning and pain during urination. He is experiencing: a. Dysuria. b. Nocturia. c. Polyuria. d. Hematuria.

ANS: A Dysuria (burning with urination) is common with acute cystitis, prostatitis, and urethritis. Nocturia is voiding during the night. Polyuria is voiding in excessive quantities. Hematuria is voiding with blood in the urine. DIF: Cognitive Level: Applying (Application) REF: p. 695 MSC: Client Needs: Physiologic Integrity: Physiologic Adaptation

During a health history of a patient who complains of chronic constipation, the patient asks the nurse about high-fiber foods. The nurse relates that an example of a high-fiber food would be: a. Broccoli. b. Hamburger. c. Iceberg lettuce. d. Yogurt.

ANS: A High-fiber foods are either soluble type (e.g., beans, prunes, barley, broccoli) or insoluble type (e.g., cereals, wheat germ). The other examples are not considered high-fiber foods. DIF: Cognitive Level: Understanding (Comprehension) REF: p. 724 MSC: Client Needs: Health Promotion and Maintenance

An older man is concerned about his sexual performance. The nurse knows that in the absence of disease, a withdrawal from sexual activity later in life may be attributable to: a. Side effects of medications. b. Decreased libido with aging. c. Decreased sperm production. d. Decreased pleasure from sexual intercourse.

ANS: A In the absence of disease, a withdrawal from sexual activity may be attributable to side effects of medications such as antihypertensives, antidepressants, sedatives, psychotropics, antispasmotics, tranquilizers or narcotics, and estrogens. The other options are not correct. DIF: Cognitive Level: Understanding (Comprehension) REF: p. 693 MSC: Client Needs: Health Promotion and Maintenance

When performing a genitourinary assessment, the nurse notices that the urethral meatus is ventrally positioned. This finding is: a. Called hypospadias. b. A result of phimosis. c. Probably due to a stricture. d. Often associated with aging.

ANS: A Normally, the urethral meatus is positioned just about centrally. Hypospadias is the ventral location of the urethral meatus. The position of the meatus does not change with aging. Phimosis is the inability to retract the foreskin. A stricture is a narrow opening of the meatus. DIF: Cognitive Level: Applying (Application) REF: p. 700 MSC: Client Needs: Safe and Effective Care Environment: Management of Care

The nurse is inspecting the scrotum and testes of a 43-year-old man. Which finding would require additional follow-up and evaluation? a. Skin on the scrotum is taut. b. Left testicle hangs lower than the right testicle. c. Scrotal skin has yellowish 1-cm nodules that are firm and nontender. d. Testes move closer to the body in response to cold temperatures.

ANS: A Scrotal swelling may cause the skin to be taut and to display pitting edema. Normal scrotal skin is rugae, and asymmetry is normal with the left scrotal half usually lower than the right. The testes may move closer to the body in response to cold temperatures. DIF: Cognitive Level: Analyzing (Analysis) REF: p. 718 MSC: Client Needs: Physiologic Integrity: Physiologic Adaptation

A male patient with possible fertility problems asks the nurse where sperm is produced. The nurse knows that sperm production occurs in the: a. Testes. b. Prostate. c. Epididymis. d. Vas deferens.

ANS: A Sperm production occurs in the testes, not in the other structures listed. DIF: Cognitive Level: Remembering (Knowledge) REF: p. 692 MSC: Client Needs: Physiologic Integrity

A 70-year-old man is visiting the clinic for difficulty in passing urine. In the health history, he indicates that he has to urinate frequently, especially at night. He has burning when he urinates and has noticed pain in his back. Considering this history, what might the nurse expect to find during the physical assessment? a. Asymmetric, hard, and fixed prostate gland b. Occult blood and perianal pain to palpation c. Symmetrically enlarged, soft prostate gland d. Soft nodule protruding from the rectal mucosa

ANS: A Subjective symptoms of carcinoma of the prostate include frequency, nocturia, hematuria, weak stream, hesitancy, pain or burning on urination, and continuous pain in lower back, pelvis, and thighs. Objective symptoms of carcinoma of the prostate include a malignant neoplasm that often starts as a single hard nodule on the posterior surface, producing asymmetry and a change in consistency. As it invades normal tissue, multiple hard nodules appear, or the entire gland feels stone hard and fixed. DIF: Cognitive Level: Analyzing (Analysis) REF: p. 735 MSC: Client Needs: Physiologic Integrity: Physiologic Adaptation

A 62-year-old man is experiencing fever, chills, malaise, urinary frequency, and urgency. He also reports urethral discharge and a dull aching pain in the perineal and rectal area. These symptoms are most consistent with which condition? a. Prostatitis b. Polyps c. Carcinoma of the prostate d. BPH

ANS: A The common presenting symptoms of prostatitis are fever, chills, malaise, and urinary frequency and urgency. The individual may also have dysuria, urethral discharge, and a dull aching pain in the perineal and rectal area. These symptoms are not consistent with polyps. (See Table 25-3 for the descriptions of carcinoma of the prostate and BPH.) DIF: Cognitive Level: Analyzing (Analysis) REF: p. 735 MSC: Client Needs: Physiologic Integrity: Physiologic Adaptation

The nurse is caring for a newborn infant. Thirty hours after birth, the infant passes a dark green meconium stool. The nurse recognizes this is important because the: a. Stool indicates anal patency. b. Dark green color indicates occult blood in the stool. c. Meconium stool can be reflective of distress in the newborn. d. Newborn should have passed the first stool within 12 hours after birth.

ANS: A The first stool passed by the newborn is dark green meconium and occurs within 24 to 48 hours of birth, indicating anal patency. The other responses are not correct. DIF: Cognitive Level: Understanding (Comprehension) REF: p. 723 MSC: Client Needs: Health Promotion and Maintenance

The nurse is preparing to palpate the rectum and should use which of these techniques? The nurse should: a. Flex the finger, and slowly insert it toward the umbilicus. b. First instruct the patient that this procedure will be painful. c. Insert an extended index finger at a right angle to the anus. d. Place the finger directly into the anus to overcome the tight sphincter.

ANS: A The nurse should gently place the pad of the index finger against the anal verge. The nurse will feel the sphincter tighten and then relax. As it relaxes, the nurse should flex the tip of the finger and slowly insert it into the anal canal in a direction toward the umbilicus. The nurse should never approach the anus at right angles with the index finger extended; doing so would cause pain. The nurse should instruct the patient that palpation is not painful but may feel like needing to move the bowels. DIF: Cognitive Level: Understanding (Comprehension) REF: p. 726 MSC: Client Needs: Safe and Effective Care Environment: Management of Care

Which of these statements is most appropriate when the nurse is obtaining a genitourinary history from an older man? a. "Do you need to get up at night to urinate?" b. "Do you experience nocturnal emissions, or 'wet dreams'?" c. "Do you know how to perform a testicular self-examination?" d. "Has anyone ever touched your genitals when you did not want them to?"

ANS: A The older male patient should be asked about the presence of nocturia. Awaking at night to urinate may be attributable to a diuretic medication, fluid retention from mild heart failure or varicose veins, or fluid ingestion 3 hours before bedtime, especially coffee and alcohol. The other questions are more appropriate for younger men. DIF: Cognitive Level: Analyzing (Analysis) REF: p. 699 MSC: Client Needs: Health Promotion and Maintenance

. During a class on cultural practices, the nurse hears the term cultural taboo. Which statement illustrates the concept of a cultural taboo? a. Believing that illness is a punishment of sin b. Trying prayer before seeking medical help c. Refusing to accept blood products as part of treatment d. Stating that a childs birth defect is the result of the parents sins

C

During a physical examination, the nurse finds that a male patient's foreskin is fixed and tight and will not retract over the glans. The nurse recognizes that this condition is: a. Phimosis. b. Epispadias. c. Urethral stricture. d. Peyronie disease.

ANS: A With phimosis, the foreskin is nonretractable, forming a pointy tip of the penis with a tiny orifice at the end of the glans. The foreskin is advanced and so tight that it is impossible to retract over the glans. This condition may be congenital or acquired from adhesions related to infection. (See Table 24-3 for information on urethral stricture. See Table 24-4 for information on epispadias and Peyronie disease.) DIF: Cognitive Level: Applying (Application) REF: p. 700 MSC: Client Needs: Physiologic Integrity: Physiologic Adaptation

The nurse is performing a digital examination of a patient's prostate gland and notices that a normal prostate gland includes which of the following characteristics? Select all that apply. a. 1 cm protrusion into the rectum b. Heart-shaped with a palpable central groove c. Flat shape with no palpable groove d. Boggy with a soft consistency e. Smooth surface, elastic, and rubbery consistency f. Fixed mobility

ANS: A, B, E The size of a normal prostate gland should be 2.5 cm long by 4 cm wide and should not protrude more than 1 cm into the rectum. The prostate should be heart-shaped, with a palpable central groove, a smooth surface, and elastic with a rubbery consistency. Abnormal findings include a flat shape with no palpable groove, boggy with a soft consistency, and fixed mobility. DIF: Cognitive Level: Analyzing (Analysis) REF: p. 728 MSC: Client Needs: Health Promotion and Maintenance

15. The nurse is assessing an elderly woman and suspects abuse. Which questions are appropriate for screening for abuse? Select all that apply. A. "Has anyone ever physically hurt you?" B. "Are you being abused?" C. "Are you alone a lot?" D. "Are you afraid of anybody at home or anyone who enters your home?" E. "Has anyone ever failed to help you take care of yourself when you needed help?"

ANS: A, C, D, E Directly asking "Are you being abused?" is not appropriate for a screening question for abuse because the woman could easily say "no" and no further information would be obtained. The other questions are among the questions recommended by the American Medical Association when screening for elder abuse.

A 16-year-old boy is brought to the clinic for a problem that he refused to let his mother see. The nurse examines him, and finds that he has scrotal swelling on the left side. He had the mumps the previous week, and the nurse suspects that he has orchitis. Which of the following assessment findings support this diagnosis? Select all that apply. a. Swollen testis b. Mass that transilluminates c. Mass that does not transilluminate d. Scrotum that is nontender upon palpation e. Scrotum that is tender upon palpation f. Scrotal skin that is reddened

ANS: A, C, E, F With orchitis, the testis is swollen, with a feeling of weight, and is tender or painful. The mass does not transilluminate, and the scrotal skin is reddened. Transillumination of a mass occurs with a hydrocele, not orchitis. DIF: Cognitive Level: Applying (Application) REF: p. 718 MSC: Client Needs: Health Promotion and Maintenance

A 55-year-old man is in the clinic for a yearly checkup. He is worried because his father died of prostate cancer. The nurse knows which tests should be performed at this time? Select all that apply. a. Blood test for prostate-specific antigen (PSA) b. Urinalysis c. Transrectal ultrasound d. Digital rectal examination (DRE) e. Prostate biopsy

ANS: A, D Prostate cancer is typically detected by testing the blood for PSA or by a DRE. It is recommended that both PSA and DRE be offered to men annually, beginning at age 50 years. If the PSA is elevated, then further laboratory work or a transrectal ultrasound (TRUS) and biopsy may be recommended. DIF: Cognitive Level: Applying (Application) REF: p. 708 MSC: Client Needs: Health Promotion and Maintenance

The nurse is reviewing concepts of cultural aspects of pain. Which statement is true regarding pain? a. All patients will behave the same way when in pain. b. Just as patients vary in their perceptions of pain, so will they vary in their expressions of pain. c. Cultural norms have very little to do with pain tolerance, because pain tolerance is always biologically determined. d. A patients expression of pain is largely dependent on the amount of tissue injury associated with the pain.

B

11. During a home visit, the nurse notices that an elderly woman has very little food in her cabinets or refrigerator and that most of her prescription bottles are empty. She says that she has enough money, but her nephew has her checkbook and he "takes care of everything." She says, "Oh, my nephew will get around to getting groceries and my medicine when he can. He's very busy." This is an example of: A. Financial abuse. B. Financial neglect. C. Psychological neglect. D. Physical abuse

ANS: B Financial neglect is failure to use the assets of the elderly person to provide services needed by the elderly person. Financial abuse is intentional misuse of the elderly person's financial or material resources without the informed consent of the person; there is no evidence of intent in this example.

13. During an interview, a woman has answered "yes" to two of the Abuse Assessment Screen questions. What should the nurse say next? A. "I need to report this abuse to the authorities." B. "Tell me about this abuse in your relationship." C. "So you were abused?" D. "Do you know what caused this abuse?"

ANS: B If a woman answers "yes" to any of the Abuse Assessment Screen questions, then the nurse should ask questions designed to assess how recent and how serious the abuse was. Asking the woman an open-ended question, such as "tell me about this abuse in your relationship" is a good way to start.

12. The nurse suspects abuse when a 10-year-old child is taken to the urgent care center for a leg injury. The best way to document the history and physical findings is to: A. Write what the child's caregiver tells the nurse. B. Use the words the child has given to describe how the injury occurred. C. Write what the nurse observes during the conversation. D. Rely on photographs of the injuries.

ANS: B When documenting the history and physical findings of child abuse and neglect, use the words the child has given to describe how his or her injury occurred. Remember that the abuser may be accompanying the child.

During a digital examination of the rectum, the nurse notices that the patient has hard feces in the rectum. The patient complains of feeling "full," has a distended abdomen, and states that she has not had a bowel movement "for several days." The nurse suspects which condition? a. Rectal polyp b. Fecal impaction c. Rectal abscess d. Rectal prolapse

ANS: B A fecal impaction is a collection of hard, desiccated feces in the rectum. The obstruction often results from decreased bowel motility, in which more water is reabsorbed from the stool. (See Table 25-2 for the descriptions of rectal polyps and abscesses; See Table 25-1 for a description of rectal prolapse.) DIF: Cognitive Level: Applying (Application) REF: p. 734 MSC: Client Needs: Physiologic Integrity: Physiologic Adaptation

A 40-year-old black man is in the office for his annual physical examination. Which statement regarding the PSA blood test is true, according to the American Cancer Society? The PSA: a. Should be performed with this visit. b. Should be performed at age 45 years. c. Should be performed at age 50 years. d. Is only necessary if a family history of prostate cancer exists.

ANS: B According to the American Cancer Society (2006), the PSA blood test should be performed annually for black men beginning at age 45 years and annually for all other men over age 50 years. DIF: Cognitive Level: Applying (Application) REF: p. 725 MSC: Client Needs: Health Promotion and Maintenance

During a discussion for a men's health group, the nurse relates that the group with the highest incidence of prostate cancer is: a. Asian Americans. b. Blacks. c. American Indians. d. Hispanics.

ANS: B According to the American Cancer Society (2010), black men have a higher rate of prostate cancer than other racial groups. DIF: Cognitive Level: Remembering (Knowledge) REF: p. 723 MSC: Client Needs: Physiologic Integrity: Reduction of Risk Potential

When performing a genital examination on a 25-year-old man, the nurse notices deeply pigmented, wrinkled scrotal skin with large sebaceous follicles. On the basis of this information, the nurse would: a. Squeeze the glans to check for the presence of discharge. b. Consider this finding as normal, and proceed with the examination. c. Assess the testicles for the presence of masses or painless lumps. d. Obtain a more detailed history, focusing on any scrotal abnormalities the patient has noticed.

ANS: B After adolescence, the scrotal skin is deeply pigmented and has large sebaceous follicles and appears corrugated. DIF: Cognitive Level: Applying (Application) REF: p. 691 MSC: Client Needs: Health Promotion and Maintenance

The nurse is reviewing the development of culture. Which statement is correct regarding the development of ones culture? Culture is: a. Genetically determined on the basis of racial background. b. Learned through language acquisition and socialization. c. A nonspecific phenomenon and is adaptive but unnecessary. d. Biologically determined on the basis of physical characteristics.

B

After completing an assessment of a 60-year-old man with a family history of colon cancer, the nurse discusses with him early detection measures for colon cancer. The nurse should mention the need for a(n): a. Annual proctoscopy. b. Colonoscopy every 10 years. c. Fecal test for blood every 6 months. d. DREs every 2 years.

ANS: B Early detection measures for colon cancer include a DRE performed annually after age 50 years, an annual fecal occult blood test after age 50 years, a sigmoidoscopic examination every 5 years or a colonoscopy every 10 years after age 50 years, and a PSA blood test annually for men over 50 years old, except beginning at age 45 years for black men (American Cancer Society, 2006). DIF: Cognitive Level: Applying (Application) REF: p. 725 MSC: Client Needs: Health Promotion and Maintenance

A 55-year-old man is experiencing severe pain of sudden onset in the scrotal area. It is somewhat relieved by elevation. On examination the nurse notices an enlarged, red scrotum that is very tender to palpation. Distinguishing the epididymis from the testis is difficult, and the scrotal skin is thick and edematous. This description is consistent with which of these? a. Varicocele b. Epididymitis c. Spermatocele d. Testicular torsion

ANS: B Epididymitis presents as severe pain of sudden onset in the scrotum that is somewhat relieved by elevation. On examination, the scrotum is enlarged, reddened, and exquisitely tender. The epididymis is enlarged and indurated and may be hard to distinguish from the testis. The overlying scrotal skin may be thick and edematous. (See Table 24-6 for more information and for the descriptions of the other terms.) DIF: Cognitive Level: Analyzing (Analysis) REF: p. 716 MSC: Client Needs: Physiologic Integrity: Physiologic Adaptation

A 30-year-old woman is visiting the clinic because of "pain in my bottom when I have a bowel movement." The nurse should assess for which problem? a. Pinworms b. Hemorrhoids c. Colon cancer d. Fecal incontinence

ANS: B Having painful bowel movements, known as dyschezia, may be attributable to a local condition (hemorrhoid or fissure) or constipation. The other responses are not correct. DIF: Cognitive Level: Applying (Application) REF: p. 723 MSC: Client Needs: Physiologic Integrity: Physiologic Adaptation

The nurse is reviewing theories of illness. The germ theory, which states that microscopic organisms such as bacteria and viruses are responsible for specific disease conditions, is a basic belief of which theory of illness? a. Holistic b. Biomedical c. Naturalistic d. Magicoreligious

B

The nurse is describing how to perform a testicular self-examination to a patient. Which statement is most appropriate? a. "A good time to examine your testicles is just before you take a shower." b. "If you notice an enlarged testicle or a painless lump, call your health care provider." c. "The testicle is egg shaped and movable. It feels firm and has a lumpy consistency." d. "Perform a testicular examination at least once a week to detect the early stages of testicular cancer."

ANS: B If the patient notices a firm painless lump, a hard area, or an overall enlarged testicle, then he should call his health care provider for further evaluation. The testicle normally feels rubbery with a smooth surface. A good time to examine the testicles is during the shower or bath, when one's hands are warm and soapy and the scrotum is warm. Testicular self-examination should be performed once a month. DIF: Cognitive Level: Applying (Application) REF: p. 704 MSC: Client Needs: Health Promotion and Maintenance

The structure that secretes a thin, milky alkaline fluid to enhance the viability of sperm is the: a. Cowper gland. b.Prostate gland. c. Median sulcus. d. Bulbourethral gland.

ANS: B In men, the prostate gland secretes a thin milky alkaline fluid that enhances sperm viability. The Cowper glands (also known as bulbourethral glands) secrete a clear, viscid mucus. The median sulcus is a groove that divides the lobes of the prostate gland and does not secrete fluid. DIF: Cognitive Level: Remembering (Knowledge) REF: p. 722 MSC: Client Needs: General

When performing a scrotal assessment, the nurse notices that the scrotal contents show a red glow with transillumination. On the basis of this finding the nurse would: a. Assess the patient for the presence of a hernia. b. Suspect the presence of serous fluid in the scrotum. c. Consider this finding normal, and proceed with the examination. d. Refer the patient for evaluation of a mass in the scrotum.

ANS: B Normal scrotal contents do not allow light to pass through the scrotum. However, serous fluid does transilluminate and shows as a red glow. Neither a mass nor a hernia would transilluminate. DIF: Cognitive Level: Analyzing (Analysis) REF: p. 703 MSC: Client Needs: Physiologic Integrity: Physiologic Adaptation

The mother of a 10-year-old boy asks the nurse to discuss the recognition of puberty. The nurse should reply by saying: a. "Puberty usually begins around 15 years of age." b. "The first sign of puberty is an enlargement of the testes." c. "The penis size does not increase until about 16 years of age." d. "The development of pubic hair precedes testicular or penis enlargement."

ANS: B Puberty begins sometime between age 9 for African Americans and age 10 for Caucasians and Hispanics. The first sign is an enlargement of the testes. Pubic hair appears next, and then penis size increases. DIF: Cognitive Level: Applying (Application) REF: p. 693 MSC: Client Needs: Health Promotion and Maintenance

When assessing the scrotum of a male patient, the nurse notices the presence of multiple firm, nontender, yellow 1-cm nodules. The nurse knows that these nodules are most likely: a. From urethritis. b. Sebaceous cysts. c. Subcutaneous plaques. d. From an inflammation of the epididymis.

ANS: B Sebaceous cysts are commonly found on the scrotum. These yellowish 1-cm nodules are firm, nontender, and often multiple. The other options are not correct. DIF: Cognitive Level: Analyzing (Analysis) REF: p. 702 MSC: Client Needs: Physiologic Integrity: Physiologic Adaptation

The nurse recognizes that an example of a person who is heritage consistent would be a: a. Woman who has adapted her clothing to the clothing style of her new country. b. Woman who follows the traditions that her mother followed regarding meals. c. Man who is not sure of his ancestors country of origin. d. Child who is not able to speak his parents native language.

B

The nurse is performing an examination of the anus and rectum. Which of these statements iscorrect and important to remember during this examination? a. The rectum is approximately 8 cm long. b. The anorectal junction cannot be palpated. c. Above the anal canal, the rectum turns anteriorly. d. No sensory nerves are in the anal canal or rectum.

ANS: B The anal columns are folds of mucosa that extend vertically down from the rectum and end in the anorectal junction. This junction is not palpable but is visible on proctoscopy. The rectum is 12 cm long; just above the anal canal, the rectum dilates and turns posteriorly. DIF: Cognitive Level: Remembering (Knowledge) REF: p. 721 MSC: Client Needs: Safe and Effective Care Environment: Management of Care

The external male genital structures include the: a. Testis. b. Scrotum. c. Epididymis. d. Vas deferens.

ANS: B The external male genital structures include the penis and scrotum. The testis, epididymis, and vas deferens are internal structures. DIF: Cognitive Level: Remembering (Knowledge) REF: p. 691 MSC: Client Needs: General

Which statement concerning the sphincters is correct? a. The internal sphincter is under voluntary control. b. The external sphincter is under voluntary control. c. Both sphincters remain slightly relaxed at all times. d. The internal sphincter surrounds the external sphincter.

ANS: B The external sphincter surrounds the internal sphincter but also has a small section overriding the tip of the internal sphincter at the opening. The external sphincter is under voluntary control. Except for the passing of feces and gas, the sphincters keep the anal canal tightly closed. DIF: Cognitive Level: Remembering (Knowledge) REF: p. 721 MSC: Client Needs: General

Which characteristic of the prostate gland would the nurse recognize as an abnormal finding while palpating the prostate gland through the rectum? a. Palpable central groove b. Tenderness to palpation c. Heart shaped d. Elastic and rubbery consistency

ANS: B The normal prostate gland should feel smooth, elastic, and rubbery; slightly movable; heart-shaped with a palpable central groove; and not be tender to palpation. DIF: Cognitive Level: Analyzing (Analysis) REF: p. 728 MSC: Client Needs: Health Promotion and Maintenance

During an examination of an aging man, the nurse recognizes that normal changes to expect would be: a. Change in scrotal color. b. Decrease in the size of the penis. c. Enlargement of the testes and scrotum. d. Increase in the number of rugae over the scrotal sac.

ANS: B When assessing the genitals of an older man, the nurse may notice thinner, graying pubic hair and a decrease in the size of the penis. The size of the testes may be decreased, they may feel less firm, and the scrotal sac is pendulous with less rugae. No change in scrotal color is observed. DIF: Cognitive Level: Applying (Application) REF: p. 708 MSC: Client Needs: Health Promotion and Maintenance

8. A female patient has denied any abuse when answering the Abuse Assessment Screen, but the nurse has noticed some other conditions that are associated with intimate partner violence. Examples of such conditions include: A. Asthma. B. Confusion. C. Depression. D. Frequent colds

ANS: C Depression is one of the conditions that is particularly associated with intimate partner violence. Abused women also have been found to have more chronic health problems, such as neurologic, gastrointestinal, and gynecologic symptoms, and chronic pain, as well as suicidality and posttraumatic stress disorder symptoms.

3. The nurse is aware that intimate partner violence (IPV) screening should occur with which situation? A. When IPV is suspected B. When a woman has an unexplained injury C. As a routine part of each health care encounter D. When there is a known history of abuse in the family

ANS: C Many nursing professional organizations have called for routine, universal screening for IPV to assist women in getting help for the problem.

14. The nurse is examining a 3-year-old child who was brought to the emergency room after a fall. Which bruise, if found, would be of most concern? A. A bruise on the knee B. A bruise on the elbow C. Bruising on the abdomen D. A bruise on the shin

ANS: C Studies have shown that children who were walking often had bruises over the bony prominences of the front of their bodies. Other studies have found that bruising in "atypical" places such as the buttocks, hands, feet and abdomen was exceedingly rare and should arouse concern.

5. Which term refers to a wound produced by the tearing or splitting of body tissue, usually from blunt impact over a bony surface? A. Abrasion B. Contusion C. Laceration D. Hematoma

ANS: C The term laceration fits this definition. An abrasion is caused by rubbing of the skin or mucous membrane. A contusion is injury to tissues without breakage of skin, and a hematoma is a localized collection of extravasated blood.

A patient who is visiting the clinic complains of having "stomach pains for 2 weeks" and describes his stools as being "soft and black" for approximately the last 10 days. He denies taking any medications. The nurse is aware that these symptoms are mostly indicative of: a. Excessive fat caused by malabsorption. b. Increased iron intake, resulting from a change in diet. c. Occult blood, resulting from gastrointestinal bleeding. d. Absent bile pigment from liver problems.

ANS: C Black stools may be tarry as a result of occult blood (melena) from gastrointestinal bleeding or nontarry from ingestion of iron medications (not diet). Excessive fat causes the stool to become frothy. The absence of bile pigment causes clay-colored stools. DIF: Cognitive Level: Analyzing (Analysis) REF: p. 724 MSC: Client Needs: Physiologic Integrity: Physiologic Adaptation

The nurse is performing a genitourinary assessment on a 50-year-old obese male laborer. On examination, the nurse notices a painless round swelling close to the pubis in the area of the internal inguinal ring that is easily reduced when the individual is supine. These findings are most consistent with a(n) ______ hernia. a. Scrotal b. Femoral c. Direct inguinal d. Indirect inguinal

ANS: C Direct inguinal hernias occur most often in men over the age of 40 years. It is an acquired weakness brought on by heavy lifting, obesity, chronic cough, or ascites. The direct inguinal hernia is usually a painless, round swelling close to the pubis in the area of the internal inguinal ring that is easily reduced when the individual is supine. (See Table 24-6 for a description of scrotal hernia. See Table 24-7 for the descriptions of femoral hernias and indirect inguinal hernias.) DIF: Cognitive Level: Analyzing (Analysis) REF: p. 719 MSC: Client Needs: Physiologic Integrity: Physiologic Adaptation

During a genital examination, the nurse notices that a male patient has clusters of small vesicles on the glans, surrounded by erythema. The nurse recognizes that these lesions are: a. Peyronie disease. b. Genital warts. c. Genital herpes. d. Syphilitic cancer.

ANS: C Genital herpes, or herpes simplex virus 2 (HSV-2), infections are indicated with clusters of small vesicles with surrounding erythema, which are often painful and erupt on the glans or foreskin. (See Table 24-4 for the descriptions of the other options.) DIF: Cognitive Level: Applying (Application) REF: p. 713 MSC: Client Needs: Physiologic Integrity: Physiologic Adaptation

An accessory glandular structure for the male genital organs is the: a. Testis. b. Scrotum. c. Prostate. d. Vas deferens.

ANS: C Glandular structures accessory to the male genital organs are the prostate, seminal vesicles, and bulbourethral glands. DIF: Cognitive Level: Remembering (Knowledge) REF: p. 691 MSC: Client Needs: General

A 2-month-old uncircumcised infant has been brought to the clinic for a well-baby checkup. How would the nurse proceed with the genital examination? a. Eliciting the cremasteric reflex is recommended. b. The glans is assessed for redness or lesions. c. Retracting the foreskin should be avoided until the infant is 3 months old. d. Any dirt or smegma that has collected under the foreskin should be noted.

ANS: C If uncircumcised, then the foreskin is normally tight during the first 3 months and should not be retracted because of the risk of tearing the membrane attaching the foreskin to the shaft. The other options are not correct. DIF: Cognitive Level: Applying (Application) REF: p. 706 MSC: Client Needs: Safe and Effective Care Environment: Management of Care

During an examination of an aging man, the nurse recognizes that normal changes to expect would be: a. Enlarged scrotal sac. b. Increased pubic hair. c. Decreased penis size. d. Increased rugae over the scrotum.

ANS: C In the aging man, the amount of pubic hair decreases, the penis size decreases, and the rugae over the scrotal sac decreases. The scrotal sac does not enlarge. DIF: Cognitive Level: Understanding (Comprehension) REF: p. 693 MSC: Client Needs: Health Promotion and Maintenance

The nurse is aware of which statement to be true regarding the incidence of testicular cancer? a. Testicular cancer is the most common cancer in men aged 30 to 50 years. b. The early symptoms of testicular cancer are pain and induration. c. Men with a history of cryptorchidism are at the greatest risk for the development of testicular cancer. d. The cure rate for testicular cancer is low.

ANS: C Men with undescended testicles (cryptorchidism) are at the greatest risk for the development of testicular cancer. The overall incidence of testicular cancer is rare. Although testicular cancer has no early symptoms, when detected early and treated before metastasizing, the cure rate is almost 100%. DIF: Cognitive Level: Understanding (Comprehension) REF: p. 707 MSC: Client Needs: Health Promotion and Maintenance

A 2-year-old boy has been diagnosed with physiologic cryptorchidism. Considering this diagnosis, during assessment the nurse will most likely observe: a. Testes that are hard and painful to palpation. b. Atrophic scrotum and a bilateral absence of the testis. c. Absence of the testis in the scrotum, but the testis can be milked down. d. Testes that migrate into the abdomen when the child squats or sits cross-legged.

ANS: C Migratory testes (physiologic cryptorchidism) are common because of the strength of the cremasteric reflex and the small mass of the prepubertal testes. The affected side has a normally developed scrotum and the testis can be milked down. The other responses are not correct. DIF: Cognitive Level: Applying (Application) REF: p. 707 MSC: Client Needs: Physiologic Integrity: Physiologic Adaptation

The nurse notices that a patient has had a pale, yellow, greasy stool, or steatorrhea, and recalls that this is caused by: a. Occult bleeding. b. Absent bile pigment. c. Increased fat content. d. Ingestion of bismuth preparations.

ANS: C Steatorrhea (pale, yellow, greasy stool) is caused by increased fat content in the stools, as in malabsorption syndrome. Occult bleeding and ingestion of bismuth products cause a black stool, and absent bile pigment causes a gray-tan stool. DIF: Cognitive Level: Understanding (Comprehension) REF: p. 729 MSC: Client Needs: Physiologic Integrity: Physiologic Adaptation

A 45-year-old mother of two children is seen at the clinic for complaints of "losing my urine when I sneeze." The nurse documents that she is experiencing: a. Urinary frequency. b. Enuresis. c. Stress incontinence. d. Urge incontinence.

ANS: C Stress incontinence is involuntary urine loss with physical strain, sneezing, or coughing that occurs as a result to weakness of the pelvic floor. Urinary frequency is urinating more times than usual (more than five to six times per day). Enuresis is involuntary passage of urine at night after age 5 to 6 years (bed wetting). Urge incontinence is involuntary urine loss from overactive detrusor muscle in the bladder. It contracts, causing an urgent need to void. DIF: Cognitive Level: Applying (Application) REF: p. 696 MSC: Client Needs: Physiologic Integrity: Physiologic Adaptation

When the nurse is performing a testicular examination on a 25-year-old man, which finding is considered normal? a. Nontender subcutaneous plaques b. Scrotal area that is dry, scaly, and nodular c. Testes that feel oval and movable and are slightly sensitive to compression d. Single, hard, circumscribed, movable mass, less than 1 cm under the surface of the testes

ANS: C Testes normally feel oval, firm and rubbery, smooth, and bilaterally equal and are freely movable and slightly tender to moderate pressure. The scrotal skin should not be dry, scaly, or nodular or contain subcutaneous plaques. Any mass would be an abnormal finding. DIF: Cognitive Level: Applying (Application) REF: p. 702 MSC: Client Needs: Safe and Effective Care Environment: Management of Care

The nurse is examining only the rectal area of a woman and should place the woman in what position? a. Lithotomy b. Prone c. Left lateral decubitus d. Bending over the table while standing

ANS: C The nurse should place the female patient in the lithotomy position if the genitalia are being examined as well. The left lateral decubitus position is used for the rectal area alone. DIF: Cognitive Level: Understanding (Comprehension) REF: p. 725 MSC: Client Needs: Safe and Effective Care Environment: Management of Care

While assessing a patient who is hospitalized and bedridden, the nurse notices that the patient has been incontinent of stool. The stool is loose and gray-tan in color. The nurse recognizes that this finding indicates which of the following? a. Occult blood b. Inflammation c. Absent bile pigment d. Ingestion of iron preparations

ANS: C The presence of gray-tan stool indicates absent bile pigment, which can occur with obstructive jaundice. The ingestion of iron preparations and the presence of occult blood turns the stools to a black color. Jellylike mucus shreds mixed in the stool would indicate inflammation. DIF: Cognitive Level: Applying (Application) REF: p. 729 MSC: Client Needs: Physiologic Integrity: Physiologic Adaptation

A 46-year-old man requires an assessment of his sigmoid colon. Which instrument or technique is most appropriate for this examination? a. Proctoscope b. Ultrasound c. Colonoscope d. Rectal examination with an examining finger

ANS: C The sigmoid colon is 40 cm long, and the nurse knows that it is accessible to examination only with the colonoscope. The other responses are not appropriate for an examination of the entire sigmoid colon. DIF: Cognitive Level: Understanding (Comprehension) REF: p. 722 MSC: Client Needs: Safe and Effective Care Environment: Management of Care

During an examination, the nurse notices that a male patient has a red, round, superficial ulcer with a yellowish serous discharge on his penis. On palpation, the nurse finds a nontender base that feels like a small button between the thumb and fingers. At this point the nurse suspects that this patient has: a. Genital warts. b. Herpes infection. c. Syphilitic chancre. d. Carcinoma lesion.

ANS: C This lesion indicates syphilitic chancre, which begins within 2 to 4 weeks of infection. (See Table 24-4 for the descriptions of the other options.) DIF: Cognitive Level: Analyzing (Analysis) REF: p. 714 MSC: Client Needs: Physiologic Integrity: Physiologic Adaptation

During an assessment of the newborn, the nurse expects to see which finding when the anal area is slightly stroked? a. Jerking of the legs b. Flexion of the knees c. Quick contraction of the sphincter d. Relaxation of the external sphincter

ANS: C To assess sphincter tone, the nurse should check the anal reflex by gently stroking the anal area and noticing a quick contraction of the sphincter. The other responses are not correct. DIF: Cognitive Level: Understanding (Comprehension) REF: p. 729 MSC: Client Needs: Health Promotion and Maintenance

Which of the following statements is true regarding the *internal structures* of the breast? The breast is: a. mainly muscle, with very little fibrous tissue. b. composed of fibrous, glandular, and adipose tissue. c. composed mostly of milk ducts, known as lactiferous ducts. d. composed of glandular tissue, which supports the breast by attaching to the chest wall.

B The breast is composed of glandular tissue, fibrous tissue (including the suspensory ligaments), and adipose tissue.

6. During an examination, the nurse notices a patterned injury on a patient's back. Which of these would cause such an injury? A. Blunt force B. Friction abrasion C. Stabbing from a kitchen knife D. Whipping from an extension cord

ANS: D A patterned injury is an injury caused by an object that leaves a distinct pattern on the skin or organ. The other actions do not cause a patterned injury.

1. As a mandatory reporter of elder abuse, which of these must be present before a nurse notifies the authorities? A. Statements from the victim B. Statements from witnesses C. Proof of abuse and/or neglect D. Suspicion of elder abuse and/or neglect

ANS: D Many health care workers are under the erroneous assumption that proof is required before notification of suspected abuse can occur. Only a suspicion of elder abuse or neglect is necessary.

4. Which statement is best for the nurse to use when preparing to administer the Abuse Assessment Screen? A. "We are required by law to ask these questions." B. "We need to talk about whether you feel you have been abused." C. "We are asking these questions because we suspect that you are being abused." D. "We ask the following questions because domestic violence is so common in our society."

ANS: D Such an introduction both alerts the woman that questions about domestic violence are coming and makes sure that the woman knows she is not being singled out for these questions.

9. The nurse is using the danger assessment (DA) tool to evaluate the risk of homicide. Which of these statements best describes its use? A. The DA tool is to be administered by law enforcement personnel. B. The DA tool should be used in every assessment of suspected abuse. C. The number of yes answers indicates the woman's understanding of her situation. D. The higher the number of yes answers, the more serious the danger of the woman's situation.

ANS: D There are no predetermined cutoff scores on the DA. The more yes answers there are, the more serious the danger of the woman's situation. The use of this tool is not limited to law enforcement personnel, and not required in every case of suspected abuse.

2. During a home visit, the nurse notices that an elderly woman is caring for her bedridden husband. The woman states that this is her duty, she does the best she can, and her children come to help when they are in town. Her husband is unable to care for himself, and she appears thin, weak, and exhausted. The nurse notices that several of his prescription medication bottles are empty. This situation is best described by the term: A. Physical abuse. B. Financial neglect. C. Psychological abuse. D. Unintentional physical neglect.

ANS: D Unintentional physical neglect may occur despite good intentions. It is the failure of the family member or caregiver to provide basic goods or services. Physical abuse is defined as violent acts that result or could result in injury, pain, impairment, or disease. Financial neglect is failure to use the assets of the elderly person to provide services needed by the elderly person. Psychological abuse is defined as behaviors that result in mental anguish.

The nurse knows that a common assessment finding in a boy younger than 2 years old is: a. Inflamed and tender spermatic cord. b. Presence of a hernia in the scrotum. c. Penis that looks large in relation to the scrotum. d. Presence of a hydrocele, or fluid in the scrotum.

ANS: D A common scrotal finding in boys younger than 2 years of age is a hydrocele, or fluid in the scrotum. The other options are not correct. DIF: Cognitive Level: Applying (Application) REF: p. 707 MSC: Client Needs: Health Promotion and Maintenance

While performing a rectal examination, the nurse notices a firm, irregularly shaped mass. What should the nurse do next? a. Continue with the examination, and document the finding in the chart. b. Instruct the patient to return for a repeat assessment in 1 month. c. Tell the patient that a mass was felt, but it is nothing to worry about. d. Report the finding, and refer the patient to a specialist for further examination.

ANS: D A firm or hard mass with an irregular shape or rolled edges may signify carcinoma. Any mass that is discovered should be promptly reported for further examination. The other responses are not correct. DIF: Cognitive Level: Applying (Application) REF: p. 734 MSC: Client Needs: Health Promotion and Maintenance

A 62-year-old man states that his physician told him that he has an "inguinal hernia." He asks the nurse to explain what a hernia is. The nurse should: a. Tell him not to worry and that most men his age develop hernias. b. Explain that a hernia is often the result of prenatal growth abnormalities. c. Refer him to his physician for additional consultation because the physician made the initial diagnosis. d. Explain that a hernia is a loop of bowel protruding through a weak spot in the abdominal muscles.

ANS: D A hernia is a loop of bowel protruding through a weak spot in the musculature. The other options are not correct responses to the patient's question. DIF: Cognitive Level: Applying (Application) REF: p. 692 MSC: Client Needs: Physiologic Integrity

During an assessment of a 20-year-old man, the nurse finds a small palpable lesion with a tuft of hair located directly over the coccyx. The nurse knows that this lesion would most likely be a: a. Rectal polyp. b. Pruritus ani. c. Carcinoma. d. Pilonidal cyst.

ANS: D A pilonidal cyst or sinus is a hair-containing cyst or sinus located in the midline over the coccyx or lower sacrum. It often opens as a dimple with a visible tuft of hair and, possibly, an erythematous halo. (See Table 25-1 for more information, and also for the description of a pruritus ani. See Table 25-2 for the descriptions of rectal polyps and carcinoma.) DIF: Cognitive Level: Applying (Application) REF: p. 732 MSC: Client Needs: Physiologic Integrity: Physiologic Adaptation

A 15-year-old boy is seen in the clinic for complaints of "dull pain and pulling" in the scrotal area. On examination, the nurse palpates a soft, irregular mass posterior to and above the testis on the left. This mass collapses when the patient is supine and refills when he is upright. This description is consistent with: a. Epididymitis. b. Spermatocele. c. Testicular torsion. d. Varicocele.

ANS: D A varicocele consists of dilated, tortuous varicose veins in the spermatic cord caused by incompetent valves within the vein. Symptoms include dull pain or a constant pulling or dragging feeling, or the individual may be asymptomatic. When palpating the mass, the examiner will feel a soft, irregular mass posterior to and above the testis that collapses when the individual is supine and refills when the individual is upright. (See Table 24-6 for more information and for the descriptions of the other options.) DIF: Cognitive Level: Analyzing (Analysis) REF: p. 717 MSC: Client Needs: Physiologic Integrity: Physiologic Adaptation

Which of these statements is true regarding the penis? a. The urethral meatus is located on the ventral side of the penis. b. The prepuce is the fold of foreskin covering the shaft of the penis. c. The penis is made up of two cylindrical columns of erectile tissue. d. The corpus spongiosum expands into a cone of erectile tissue called the glans.

ANS: D At the distal end of the shaft, the corpus spongiosum expands into a cone of erectile tissue, the glans. The penis is made up of three cylindrical columns of erectile tissue. The skin that covers the glans of the penis is the prepuce. The urethral meatus forms at the tip of the glans. DIF: Cognitive Level: Remembering (Knowledge) REF: p. 691 MSC: Client Needs: General

During a health history, a patient tells the nurse that he has trouble in starting his urine stream. This problem is known as: a. Urgency. b. Dribbling. c. Frequency. d. Hesitancy.

ANS: D Hesitancy is trouble in starting the urine stream. Urgency is the feeling that one cannot wait to urinate. Dribbling is the last of the urine before or after the main act of urination. Frequency is urinating more often than usual. DIF: Cognitive Level: Understanding (Comprehension) REF: p. 696 MSC: Client Needs: Physiologic Integrity: Physiologic Adaptation

The nurse is performing a genital examination on a male patient and notices urethral drainage. When collecting urethral discharge for microscopic examination and culture, the nurse should: a. Ask the patient to urinate into a sterile cup. b. Ask the patient to obtain a specimen of semen. c. Insert a cotton-tipped applicator into the urethra. d. Compress the glans between the examiner's thumb and forefinger, and collect any discharge.

ANS: D If urethral discharge is noticed, then the examiner should collect a smear for microscopic examination and culture by compressing the glans anteroposteriorly between the thumb and forefinger. The other options are not correct actions. DIF: Cognitive Level: Applying (Application) REF: p. 701 MSC: Client Needs: Safe and Effective Care Environment: Management of Care

During an examination, the nurse asks the patient to perform the Valsalva maneuver and notices that the patient has a moist, red, doughnut-shaped protrusion from the anus. The nurse knows that this finding is consistent with a: a. Rectal polyp. b. Hemorrhoid. c. Rectal fissure. d. Rectal prolapse.

ANS: D In rectal prolapse, the rectal mucous membrane protrudes through the anus, appearing as a moist red doughnut with radiating lines. It occurs after a Valsalva maneuver, such as straining at passing stool or with exercising (see Table 25-1). (See Table 25-2 for a description of rectal polyps and Table 25-1 for the descriptions of a rectal fissure and hemorrhoids.) DIF: Cognitive Level: Understanding (Comprehension) REF: p. 733 MSC: Client Needs: Physiologic Integrity: Physiologic Adaptation

The nurse is providing patient teaching about an erectile dysfunction drug. One of the drug's potential side effects is prolonged, painful erection of the penis without sexual stimulation, which is known as: a. Orchitis. b. Stricture. c. Phimosis. d. Priapism.

ANS: D Priapism is prolonged, painful erection of the penis without sexual desire. Orchitis is inflammation of the testes. Stricture is a narrowing of the opening of the urethral meatus. Phimosis is the inability to retract the foreskin. DIF: Cognitive Level: Remembering (Knowledge) REF: p. 714 MSC: Client Needs: Physiologic Integrity: Physiologic Adaptation

Which statement concerning the anal canal is true? The anal canal: a. Is approximately 2 cm long in the adult. b. Slants backward toward the sacrum. c. Contains hair and sebaceous glands. d. Is the outlet for the gastrointestinal tract.

ANS: D The anal canal is the outlet for the gastrointestinal tract and is approximately 3.8 cm long in the adult. It is lined with a modified skin that does not contain hair or sebaceous glands, and it slants forward toward the umbilicus. DIF: Cognitive Level: Remembering (Knowledge) REF: p. 721 MSC: Client Needs: General

While performing an assessment of the perianal area of a patient, the nurse notices that the pigmentation of anus is darker than the surrounding skin, the anal opening is closed, and a skin sac that is shiny and blue is noted. The patient mentioned that he has had pain with bowel movements and has occasionally noted some spots of blood. What would this assessment and history most likely indicate? a. Anal fistula b. Pilonidal cyst c. Rectal prolapse d. Thrombosed hemorrhoid

ANS: D The anus normally looks moist and hairless, with coarse folded skin that is more pigmented than the perianal skin, and the anal opening is tightly closed. The shiny blue skin sac indicates a thrombosed hemorrhoid. DIF: Cognitive Level: Analyzing (Analysis) REF: p. 726 MSC: Client Needs: Physiologic Integrity: Physiologic Adaptation

The nurse is examining the glans and knows which finding is normal for this area? a. The meatus may have a slight discharge when the glans is compressed. b. Hair is without pest inhabitants. c. The skin is wrinkled and without lesions. d. Smegma may be present under the foreskin of an uncircumcised male.

ANS: D The glans looks smooth and without lesions and does not have hair. The meatus should not have any discharge when the glans is compressed. Some cheesy smegma may have collected under the foreskin of an uncircumcised male. DIF: Cognitive Level: Understanding (Comprehension) REF: p. 701 MSC: Client Needs: Safe and Effective Care Environment: Management of Care

During the assessment of an 18-month-old infant, the mother expresses concern to the nurse about the infant's inability to toilet train. What would be the nurse's best response? a. "Some children are just more difficult to train, so I wouldn't worry about it yet." b. "Have you considered reading any of the books on toilet training? They can be very helpful." c. "This could mean that there is a problem in your baby's development. We'll watch her closely for the next few months." d. "The nerves that will allow your baby to have control over the passing of stools are not developed until at least 18 to 24 months of age."

ANS: D The infant passes stools by reflex. Voluntary control of the external anal sphincter cannot occur until the nerves supplying the area have become fully myelinated, usually around 1 to 2 years of age. Toilet training usually starts after the age of 2 years. DIF: Cognitive Level: Applying (Application) REF: p. 723 MSC: Client Needs: Health Promotion and Maintenance

When the nurse is conducting sexual history from a male adolescent, which statement would be most appropriate to use at the beginning of the interview? a. "Do you use condoms?" b. "You don't masturbate, do you?" c. "Have you had sex in the last 6 months?" d. "Often adolescents your age have questions about sexual activity."

ANS: D The interview should begin with a permission statement, which conveys that it is normal and acceptable to think or feel a certain way. Sounding judgmental should be avoided. DIF: Cognitive Level: Analyzing (Analysis) REF: p. 698 MSC: Client Needs: Health Promotion and Maintenance

When performing a genital assessment on a middle-aged man, the nurse notices multiple soft, moist, painless papules in the shape of cauliflower-like patches scattered across the shaft of the penis. These lesions are characteristic of: a. Carcinoma. b. Syphilitic chancres. c. Genital herpes. d. Genital warts.

ANS: D The lesions of genital warts are soft, pointed, moist, fleshy, painless papules that may be single or multiple in a cauliflower-like patch. They occur on the shaft of the penis, behind the corona, or around the anus, where they may grow into large grapelike clusters. (See Table 24-4 for more information and for the descriptions of the other options.) DIF: Cognitive Level: Analyzing (Analysis) REF: p. 713 MSC: Client Needs: Physiologic Integrity: Physiologic Adaptation

A 13-year-old girl is visiting the clinic for a sports physical examination. The nurse should remember to include which of these tests in the examination? a. Testing for occult blood b. Valsalva maneuver c. Internal palpation of the anus d. Inspection of the perianal area

ANS: D The perianal region of the school-aged child and adolescent should be inspected during the examination of the genitalia. Internal palpation is not routinely performed at this age. Testing for occult blood and performing the Valsalva maneuver are also not necessary. DIF: Cognitive Level: Applying (Application) REF: p. 729 MSC: Client Needs: Health Promotion and Maintenance

A 60-year-old man has just been told that he has benign prostatic hypertrophy (BPH). He has a friend who just died from cancer of the prostate. He is concerned this will happen to him. How should the nurse respond? a. "The swelling in your prostate is only temporary and will go away." b. "We will treat you with chemotherapy so we can control the cancer." c. "It would be very unusual for a man your age to have cancer of the prostate." d. "The enlargement of your prostate is caused by hormonal changes, and not cancer."

ANS: D The prostate gland commonly starts to enlarge during the middle adult years. BPH is present in 1 in 10 men at the age of 40 years and increases with age. It is believed that the hypertrophy is caused by hormonal imbalance that leads to the proliferation of benign adenomas. The other responses are not appropriate. DIF: Cognitive Level: Applying (Application) REF: p. 723 MSC: Client Needs: Health Promotion and Maintenance

During the taking of a health history, the patient states, "It really hurts back there, and sometimes it itches, too. I have even seen blood on the tissue when I have a bowel movement. Is there something there?" The nurse should expect to see which of these upon examination of the anus? a. Rectal prolapse b. Internal hemorrhoid c. External hemorrhoid that has resolved d. External hemorrhoid that is thrombosed

ANS: D These symptoms are consistent with an external hemorrhoid. An external hemorrhoid, when thrombosed, contains clotted blood and becomes a painful, swollen, shiny blue mass that itches and bleeds with defecation. When the external hemorrhoid resolves, it leaves a flabby, painless skin sac around the anal orifice. An internal hemorrhoid is not palpable but may appear as a red mucosal mass when the person performs a Valsalva maneuver. A rectal prolapse appears as a moist, red doughnut with radiating lines. DIF: Cognitive Level: Applying (Application) REF: p. 732 MSC: Client Needs: Physiologic Integrity: Physiologic Adaptation

Which statement concerning the testes is true? a. The lymphatic vessels of the testes drain into the abdominal lymph nodes. b. The vas deferens is located along the inferior portion of each testis. c. The right testis is lower than the left because the right spermatic cord is longer. d. The cremaster muscle contracts in response to cold and draws the testicles closer to the body.

ANS: D When it is cold, the cremaster muscle contracts, which raises the scrotal sac and brings the testes closer to the body to absorb heat necessary for sperm viability. The lymphatic vessels of the testes drain into the inguinal lymph nodes. The vas deferens is located along the upper portion of each testis. The left testis is lower than the right because the left spermatic cord is longer. DIF: Cognitive Level: Remembering (Knowledge) REF: p. 691 MSC: Client Needs: General

When the nurse is performing a genital examination on a male patient, which action iscorrect? a. Auscultating for the presence of a bruit over the scrotum b. Palpating for the vertical chain of lymph nodes along the groin, inferior to the inguinal ligament c. Palpating the inguinal canal only if a bulge is present in the inguinal region during inspection d. Having the patient shift his weight onto the left (unexamined) leg when palpating for a hernia on the right side

ANS: D When palpating for the presence of a hernia on the right side, the male patient is asked to shift his weight onto the left (unexamined) leg. Auscultating for a bruit over the scrotum is not appropriate. When palpating for lymph nodes, the horizontal chain is palpated. The inguinal canal should be palpated whether a bulge is present or not. DIF: Cognitive Level: Applying (Application) REF: p. 706 MSC: Client Needs: Safe and Effective Care Environment: Management of Care

When the nurse is performing a genital examination on a male patient, the patient has an erection. The nurse's most appropriate action or response is to: a. Ask the patient if he would like someone else to examine him. b. Continue with the examination as though nothing has happened. c. Stop the examination, leave the room while stating that the examination will resume at a later time. d. Reassure the patient that this is a normal response and continue with the examination.

ANS: D When the male patient has an erection, the nurse should reassure the patient that this is a normal physiologic response to touch and proceed with the rest of the examination. The other responses are not correct and may be perceived as judgmental. DIF: Cognitive Level: Applying (Application) REF: p. 699 MSC: Client Needs: Psychosocial Integrity

When performing a genitourinary assessment on a 16-year-old male adolescent, the nurse notices a swelling in the scrotum that increases with increased intra-abdominal pressure and decreases when he is lying down. The patient complains of pain when straining. The nurse knows that this description is most consistent with a(n) ______ hernia. a. Femoral b. Incisional c. Direct inguinal d. Indirect inguinal

ANS: D With indirect inguinal hernias, pain occurs with straining and a soft swelling increases with increased intra-abdominal pressure, which may decrease when the patient lies down. These findings do not describe the other hernias. (See Table 24-7 for the descriptions of femoral, direct inguinal, and indirect inguinal hernias.) DIF: Cognitive Level: Analyzing (Analysis) REF: p. 719 MSC: Client Needs: Physiologic Integrity: Physiologic Adaptation

A 50-year-old woman with elevated serum, total cholesterol, and triglyceride levels is visiting the clinic today to find out about her laboratory results. What would be important for the nurse to include in patient teaching in relation to these tests? 1.The risks of undernutrition 2.Methods to reduce stress in her life 3.Information regarding a low saturated fat diet 4.The fact that this is hereditary and there is nothing she can do to change the levels

ANS:3 Total cholesterol is measured to evaluate fat metabolism and to assess the risk of cardiovascular disease. Serum triglycerides are used to screen for hyperlipidemia and to determine the risk of coronary artery disease.

a

An extremely important part of the history and examination in situations of intimate partner violence or older adult abuse is the a. mental status examination. b. family genogram. c. history of the present illness. d. skin assessment.

Deviation in the nipple point

And underline cancer causes fibrosis in the mammary ducts, which pulls the nipple angle towards it. Here note the swelling behind the right nipple and the nipple that tilts laterally.

Tanner stage 4 girls

Areola and nipple projects as secondary mound Pubic hair is more coarse - no triangle yet

An individual who takes the magicoreligious perspective of illness and disease is likely to believe that his or her illness was caused by: a. Germs and viruses. b. Supernatural forces. c. Eating imbalanced foods. d. An imbalance within his or her spiritual nature.

B

In the majority culture of America, coughing, sweating, and diarrhea are symptoms of an illness. For some individuals of Mexican-American origin, however, these symptoms are a normal part of living. The nurse recognizes that this difference is true, probably because Mexican-Americans: a. Have less efficient immune systems and are often ill. b. Consider these symptoms part of normal living, not symptoms of ill health. c. Come from Mexico, and coughing is normal and healthy there. d. Are usually in a lower socioeconomic group and are more likely to be sick

B

Symptoms, such as pain, are often influenced by a persons cultural heritage. Which of the following is a true statement regarding pain? a. Nurses attitudes toward their patients pain are unrelated to their own experiences with pain. b. Nurses need to recognize that many cultures practice silent suffering as a response to pain. c. A nurses area of clinical practice will most likely determine his or her assessment of a patients pain. d. A nurses years of clinical experience and current position are strong indicators of his or her response to patient pain.

B

During a breast health interview, a patient states that she has noticed pain in her left breast. The nurse's most appropriate response to this would be: a. "Don't worry about the pain; breast cancer is not painful." b. "I would like some more information about the pain in your left breast." c. "Oh, I had pain like that after my son was born; it turned out to be a blocked milk duct." d. "Breast pain is almost always the result of benign breast disease."

B Breast pain occurs with trauma, inflammation, infection, or benign breast disease. The nurse will need to gather more information about the patient's pain rather than make statements that ignore the patient's concerns.

During an interview, a patient reveals that she is pregnant. She states that she is not sure whether she will breastfeed her baby and asks for some information about this. Which of these statements by the nurse is accurate with regard to breastfeeding? a. "Breastfed babies tend to be more colicky." b. "Breastfeeding provides the perfect food and antibodies for your baby." c. "Breastfed babies eat more often than infants on formula." d. "Breastfeeding is second nature and every woman can do it."

B Exclusively breastfeeding for 6 months provides the perfect food and antibodies for the baby, decreases the risk of ear infections, promotes bonding, and provides relaxation.

A patient states during the interview that she noticed a new lump in the shower a few days ago. It was on her left breast near her axilla. The nurse should plan to: a. palpate the lump first. b. palpate the unaffected breast first. c. avoid palpating the lump because it could be a cyst, which might rupture. d. palpate the breast with the lump first but plan to palpate the axilla last.

B If the woman mentions a breast lump she has discovered herself, the nurse should examine the unaffected breast first to learn a baseline of normal consistency for this individual.

While inspecting a patient's breasts, the nurse finds that the left breast is slightly larger than the right with the presence of Montgomery's glands bilaterally. The nurse should: a. palpate over the Montgomery's glands, checking for drainage. b. consider these normal findings and proceed with the examination. c. ask extensive history questions regarding the woman's breast asymmetry. d. continue with examination and then refer the patient for further evaluation of the Montgomery's glands.

B Normal findings of the breast include one breast (most often the left) slightly larger than the other and the presence of Montgomery's glands across the areola.

During an examination of a 7-year-old girl, the nurse notices that the girl is showing breast budding. What should the nurse do next? a. Ask her if her periods have started. b. Assess the girl's weight and body mass index (BMI). c. Ask the girl's mother at what age she started to develop breasts. d. Nothing; this is a normal finding.

B Research has shown that girls with overweight or obese BMI levels have a higher occurrence of early onset of breast budding (before age 8 years for African-American girls and age 10 years for white girls) and early menarche.

If a patient reports a recent breast infection, then the nurse should expect to find _____ node enlargement. a. nonspecific b. ipsilateral axillary c. contralateral axillary d. inguinal and cervical

B The breast has extensive lymphatic drainage. Most of the lymph, more than 75%, drains into the ipsilateral, or same side, axillary nodes.

During a breast examination on a female patient, the nurse notices that the nipple is flat, broad, and fixed. The patient states it "started doing that a few months ago." This finding suggests: a. dimpling. b. a retracted nipple. c. nipple inversion. d. deviation in nipple pointing.

B The retracted nipple looks flatter and broader, like an underlying crater. A recent retraction suggests cancer, which causes fibrosis of the whole duct system and pulls in the nipple. It also may occur with benign lesions such as ectasia of the ducts. The nurse should not confuse retraction with the normal long-standing type of nipple inversion, which has no broadening and is not fixed.

In performing a breast examination, the nurse knows that it is especially important to examine the upper outer quadrant of the breast. The reason for this is that the upper outer quadrant is: a. the largest quadrant of the breast. b. the location of most breast tumors. c. where most of the suspensory ligaments attach. d. more prone to injury and calcifications than other locations in the breast.

B The upper outer quadrant is the site of most breast tumors. In the upper outer quadrant, the nurse should notice the axillary tail of Spence, the cone-shaped breast tissue that projects up into the axilla, close to the pectoral group of axillary lymph nodes.

9. A 54-year-old woman who has just completed menopause is in the clinic today for a yearly physical examination. Which of these statements should the nurse include in patient education? "A postmenopausal woman: a. Is not at any greater risk for heart disease than a younger woman." b. Should be aware that she is at increased risk for dyspareunia because of decreased vaginal secretions." c. Has only stopped menstruating; there really are no other significant changes with which she should be concerned." d. Is likely to have difficulty with sexual pleasure as a result of drastic changes in the female sexual response cycle."

B Decreased vaginal secretions leave the vagina dry and at risk for irritation and pain with intercourse (dyspareunia). The other statements are incorrect. DIF: Cognitive Level: Applying (Application) REF: p. 740 MSC: Client Needs: Health Promotion and Maintenance

35. The nurse is preparing to examine the external genitalia of a school-age girl. Which position would be most appropriate in this situation? a. In the parent's lap b. In a frog-leg position on the examining table c. In the lithotomy position with the feet in stirrups d. Lying flat on the examining table with legs extended

B For school-age children, placing them on the examining table in a frog-leg position is best. With toddlers and preschoolers, having the child on the parent's lap in a frog-leg position is best. DIF: Cognitive Level: Applying (Application) REF: p. 759 MSC: Client Needs: Health Promotion and Maintenance

45. A woman has just been diagnosed with HPV or genital warts. The nurse should counsel her to receive regular examinations because this virus makes her at a higher risk for _______ cancer. a. Uterine b. Cervical c. Ovarian d. Endometrial

B HPV is the virus responsible for most cases of cervical cancer, not the other options. DIF: Cognitive Level: Applying (Application) REF: p. 761 MSC: Client Needs: Health Promotion and Maintenance

20. A nurse is assessing a patient's risk of contracting a sexually transmitted infection (STI). An appropriate question to ask would be: a. "You know that it's important to use condoms for protection, right?" b. "Do you use a condom with each episode of sexual intercourse?" c. "Do you have a sexually transmitted infection?" d. "You are aware of the dangers of unprotected sex, aren't you?"

B In reviewing a patient's risk for STIs, the nurse should ask in a nonconfrontational manner whether condoms are being used during each episode of sexual intercourse. Asking a person whether he or she has an infection does not address the risk. DIF: Cognitive Level: Understanding (Comprehension) REF: p. 743 MSC: Client Needs: Physiologic Integrity: Reduction of Risk Potential

19. A married couple has come to the clinic seeking advice on pregnancy. They have been trying to conceive for 4 months and have not been successful. What should the nurse do first? a. Ascertain whether either of them has been using broad-spectrum antibiotics. b. Explain that couples are considered infertile after 1 year of unprotected intercourse. c. Immediately refer the woman to an expert in pelvic inflammatory disease—the most common cause of infertility. d. Explain that couples are considered infertile after 3 months of engaging in unprotected intercourse and that they will need a referral to a fertility expert.

B Infertility is considered after 1 year of engaging in unprotected sexual intercourse without conceiving. The other actions are not appropriate. DIF: Cognitive Level: Applying (Application) REF: p. 743 MSC: Client Needs: Psychosocial Integrity

26. The nurse has just completed an inspection of a nulliparous woman's external genitalia. Which of these would be a description of a finding within normal limits? a. Redness of the labia majora b. Multiple nontender sebaceous cysts c. Discharge that is foul smelling and irritating d. Gaping and slightly shriveled labia majora

B No lesions should be noted, except for the occasional sebaceous cysts, which are yellowish 1-cm nodules that are firm, nontender, and often multiple. The labia majora are dark pink, moist, and symmetric; redness indicates inflammation or lesions. Discharge that is foul smelling and irritating may indicate infection. In the nulliparous woman, the labia majora meet in the midline, are symmetric and plump. DIF: Cognitive Level: Applying (Application) REF: p. 747 MSC: Client Needs: Safe and Effective Care Environment: Management of Care

33. The nurse is palpating a female patient's adnexa. The findings include a firm, smooth uterine wall; the ovaries are palpable and feel smooth and firm. The fallopian tube is firm and pulsating. The nurse's most appropriate course of action would be to: a. Tell the patient that her examination is normal. b. Give her an immediate referral to a gynecologist. c. Suggest that she return in a month for a recheck to verify the findings. d. Tell the patient that she may have an ovarian cyst that should be evaluated further.

B Normally, the uterine wall feels firm and smooth, with the contour of the fundus rounded. Ovaries are not often palpable, but when they are, they normally feel smooth, firm, and almond shaped and are highly movable, sliding through the fingers. The fallopian tube is not normally palpable. No other mass or pulsation should be felt. Pulsation or palpable fallopian tube suggests ectopic pregnancy, which warrants immediate referral. DIF: Cognitive Level: Applying (Application) REF: p. 757 MSC: Client Needs: Health Promotion and Maintenance

13. A 50-year-old woman calls the clinic because she has noticed some changes in her body and breasts and wonders if these changes could be attributable to the hormone replacement therapy (HRT) she started 3 months earlier. The nurse should tell her: a. "HRT is at such a low dose that side effects are very unusual." b. "HRT has several side effects, including fluid retention, breast tenderness, and vaginal bleeding." c. "Vaginal bleeding with HRT is very unusual; I suggest you come into the clinic immediately to have this evaluated." d. "It sounds as if your dose of estrogen is too high; I think you may need to decrease the amount you are taking and then call back in a week."

B Side effects of HRT include fluid retention, breast pain, and vaginal bleeding. The other responses are not correct. DIF: Cognitive Level: Applying (Application) REF: p. 741 MSC: Client Needs: Physiologic Integrity: Pharmacologic and Parenteral Therapies

14. A 52-year-old patient states that when she sneezes or coughs she "wets herself a little." She is very concerned that something may be wrong with her. The nurse suspects that the problem is: a. Dysuria. b. Stress incontinence. c. Hematuria. d. Urge incontinence.

B Stress incontinence is involuntary urine loss with physical strain, sneezing, or coughing. Dysuria is pain or burning with urination. Hematuria is bleeding with urination. Urge incontinence is involuntary urine loss that occurs as a result of an overactive detrusor muscle in the bladder that contracts and causes an urgent need to void. DIF: Cognitive Level: Understanding (Comprehension) REF: p. 742 MSC: Client Needs: Physiologic Integrity: Physiologic Adaptation

44. During an examination, the nurse would expect the cervical os of a woman who has never had children to appear: a. Stellate. b. Small and round. c. As a horizontal irregular slit. d. Everted.

B The cervical os in a nulliparous woman is small and round. In the parous woman, it is a horizontal, irregular slit that also may show healed lacerations on the sides (see Figure 26-13). DIF: Cognitive Level: Understanding (Comprehension) REF: p. 750 MSC: Client Needs: Health Promotion and Maintenance

46. During an internal examination, the nurse notices that the cervix bulges outside the introitus when the patient is asked to strain. The nurse will document this as: a. Uterine prolapse, graded first degree. b. Uterine prolapse, graded second degree. c. Uterine prolapse, graded third degree. d. A normal finding.

B The cervix should not be found to bulge into the vagina. Uterine prolapse is graded as follows: first degree—the cervix appears at the introitus with straining; second degree—the cervix bulges outside the introitus with straining; and third degree—the whole uterus protrudes, even without straining (essentially, the uterus is inside out). DIF: Cognitive Level: Applying (Application) REF: p. 766 MSC: Client Needs: Physiologic Integrity: Physiologic Adaptation

22. When the nurse is discussing sexuality and sexual issues with an adolescent, a permission statement helps convey that it is normal to think or feel a certain way. Which statement is the best example of a permission statement? a. "It is okay that you have become sexually active." b. "Girls your age often have questions about sexual activity. Do you have any questions?" c. "If it is okay with you, I'd like to ask you some questions about your sexual history." d. "Girls your age often engage in sexual activities. It is okay to tell me if you have had intercourse."

B The examiner should start with a permission statement such as, "Girls your age often experience ..." A permission statement conveys the idea that it is normal to think or feel a certain way, and implying that the topic is normal and unexceptional is important. DIF: Cognitive Level: Understanding (Comprehension) REF: p. 743 MSC: Client Needs: Psychosocial Integrity

24. During the examination portion of a patient's visit, she will be in lithotomy position. Which statement reflects some things that the nurse can do to make this position more comfortable for her? a. Ask her to place her hands and arms over her head. b. Elevate her head and shoulders to maintain eye contact. c. Allow her to choose to have her feet in the stirrups or have them resting side by side on the edge of the table. d. Allow her to keep her buttocks approximately 6 inches from the edge of the table to prevent her from feeling as if she will fall off.

B The nurse should elevate her head and shoulders to maintain eye contact. The patient's arms should be placed at her sides or across the chest. Placing her hands and arms over her head only tightens the abdominal muscles. The feet should be placed into the stirrups, knees apart, and buttocks at the edge of the examining table. The stirrups are placed so that the legs are not abducted too far. DIF: Cognitive Level: Applying (Application) REF: p. 745 MSC: Client Needs: Safe and Effective Care Environment: Management of Care

30. A patient calls the clinic for instructions before having a Papanicolaou (Pap) smear. The most appropriate instructions from the nurse are: a. "If you are menstruating, please use pads to avoid placing anything into the vagina." b. "Avoid intercourse, inserting anything into the vagina, or douching within 24 hours of your appointment." c. "If you suspect that you have a vaginal infection, please gather a sample of the discharge to bring with you." d. "We would like you to use a mild saline douche before your examination. You may pick this up in our office."

B When instructing a patient before Pap smear is obtained, the nurse should follow these guidelines: Do not obtain during the woman's menses or if a heavy infectious discharge is present. Instruct the woman not to douche, have intercourse, or put anything into the vagina within 24 hours before collecting the specimens. Any specimens will be obtained during the visit, not beforehand. DIF: Cognitive Level: Applying (Application) REF: p. 752 MSC: Client Needs: Health Promotion and Maintenance

1. The nurse is palpating an ovarian mass during an internal examination of a 63-year-old woman. Which findings of the mass's characteristics would suggest the presence of an ovarian cyst? Select all that apply. a. Heavy and solid b. Mobile and fluctuant c. Mobile and solid d. Fixed e. Smooth and round f. Poorly defined

B, E An ovarian cyst (fluctuant ovarian mass) is usually asymptomatic and would feel like a smooth, round, fluctuant, mobile, nontender mass on the ovary. A mass that is heavy, solid, fixed, and poorly defined suggests malignancy. A benign mass may feel mobile and solid. DIF: Cognitive Level: Applying (Application) REF: p. 722 MSC: Client Needs: Physiologic Integrity: Physiologic Adaptation

Fibroadenoma of breast

Benign tumor's, most commonly present as self detected in late adolescence. Solitary nontender mass that is solid, firm, robbery and elastic. Round oval or lobular 1 to 5 cm. It is freely movable slippery. Fingers slide easily through tissue. Diagnosed by triple test palpation, ultrasound, needle biopsy.

carcinoma of the breast (nipple)

Bloody nipple discharge that is unilateral and from a single that requires further investigation.

Tanner Stage 3 girls

Breast an Ariola and large, the nipple is flush with the breast surface

Tanner stage 2 girls

Breast buds with areolar enlargement Pubic hair sparse

b

Bruising on a nonwalking or noncruising child: a. Is a common finding from normal infant activity b. Needs to be further evaluated for either an abusive or medical explanation c. Is commonly seen on the buttocks d. Cannot be reported until after a full medical evaluation

After a class on culture and ethnicity, the new graduate nurse reflects a correct understanding of the concept of ethnicity with which statement? a. Ethnicity is dynamic and ever changing. b. Ethnicity is the belief in a higher power. c. Ethnicity pertains to a social group within the social system that claims shared values and traditions. d. Ethnicity is learned from birth through the processes of language acquisition and socialization.

C

An older Mexican-American woman with traditional beliefs has been admitted to an inpatient care unit. A culturally sensitive nurse would: a. Contact the hospital administrator about the best course of action. b. Automatically get a curandero for her, because requesting one herself is not culturally appropriate. c. Further assess the patients cultural beliefs and offer the patient assistance in contacting a curandero or priest if she desires. d. Ask the family what they would like to do because Mexican-Americans traditionally give control of decision making to their families.

C

During a seminar on cultural aspects of nursing, the nurse recognizes that the definition stating the specific and distinct knowledge, beliefs, skills, and customs acquired by members of a society reflects which term? a. Mores b. Norms c. Culture d. Social learning

C

If an American Indian woman has come to the clinic to seek help with regulating her diabetes, then the nurse can expect that she: a. Will comply with the treatment prescribed. b. Has obviously given up her belief in naturalistic causes of disease. c. May also be seeking the assistance of a shaman or medicine man. d. Will need extra help in dealing with her illness and may be experiencing a crisis of faith.

C

Many Asians believe in the yin/yang theory, which is rooted in the ancient Chinese philosophy of Tao. Which statement most accurately reflects health in an Asian with this belief? a. A person is able to work and produce. b. A person is happy, stable, and feels good. c. All aspects of the person are in perfect balance. d. A person is able to care for others and function socially.

C

The nurse is comparing the concepts of religion and spirituality. Which of the following is an appropriate component of ones spirituality? a. Belief in and the worship of God or gods b. Attendance at a specific church or place of worship c. Personal effort made to find purpose and meaning in life d. Being closely tied to ones ethnic background

C

The nurse is preparing for a class in early detection of breast cancer. Which statement is true with regard to breast cancer in African-American women in the United States? a. Breast cancer is not a threat to African-American women. b. African-American women have a lower incidence of regional or distant breast cancer than white women. c. African-American women are more likely to die of breast cancer at any age. d. Breast cancer incidence in African-American women is higher than that of white women after age 45.

C African-American women have a higher incidence of breast cancer before age 45 years than white women, and are more likely to die of their disease. In addition, African-American women are significantly more likely to be diagnosed with regional or distant breast cancer than are white women. This racial difference in mortality rates may be related to insufficient use of screening measures and lack of access to health care.

In examining a 70-year-old male patient, the nurse notices that he has bilateral gynecomastia. Which of the following describes the nurse's best course of action? a. Recommend that he make an appointment with his physician for a mammogram. b. Ignore it; it is not unusual for men to have benign breast enlargement. c. Explain that this condition may be the result of hormonal changes and recommend that he see his physician. d. Tell him that gynecomastia in men is usually associated with prostate enlargement and recommend that he be screened thoroughly.

C Gynecomastia may reappear in the aging male and may be due to testosterone deficiency.

The nurse has palpated a lump in a female patient's right breast. The nurse documents this as a small, round, firm, distinct, lump located at 2 o'clock, 2 cm from the nipple. It is nontender and fixed. There is no associated retraction of skin or nipple, no erythema, and no axillary lymphadenopathy. Which of these statements reveals the information that is missing from the documentation? It is missing information about: a. the shape of the lump. b. the lump's consistency. c. the size of the lump. d. whether the lump is solitary or multiple.

C If the nurse feels a lump or mass, he or she should note these characteristics: (1) location, (2) size-judge in centimeters in three dimensions: width x length x thickness, (3) shape, (4) consistency, (5) motility, (6) distinctness, (7) nipple, (8) the skin over the lump, (9) tenderness, and (10) lymphadenopathy.

During a physical examination, a 45-year-old woman states that she has had a crusty, itchy rash on her breast for about 2 weeks. In trying to find the cause of the rash, which of these would be important for the nurse to determine? a. Is the rash raised and red? b. Does it appear to be cyclic? c. Where did it first appear-on the nipple, the areola, or the surrounding skin? d. What was she doing when she first noticed the rash, and do her actions make it worse?

C It is important for the nurse to determine where the rash first appeared. Paget's disease starts with a small crust on the nipple apex and then spreads to the areola. Eczema or other dermatitis rarely starts at nipple unless it results from breastfeeding. It usually starts on the areola or surrounding skin and then spreads to the nipple. See Table 17-6.

A patient contacts the office and tells the nurse that she is worried about her 10-year-old daughter having breast cancer. She describes a unilateral enlargement of the right breast with associated tenderness. She is worried because the left breast is not enlarged. What would be the nurse's best response? a. Tell the mother that breast development is usually fairly symmetric and she should be examined right away. b. Tell the mother that she should bring her daughter in right away because breast cancer is fairly common in preadolescent girls. c. Tell the mother that, although an examination of her daughter would rule out a problem, it is most likely normal breast development. d. Tell the mother that it is unusual for breasts that are first developing to feel tender because they haven't developed much fibrous tissue.

C Occasionally one breast may grow faster than the other, producing a temporary asymmetry. This may cause some distress; reassurance is necessary. Tenderness is common also.

A 54-year-old man comes to the clinic with a "horrible problem." He tells the nurse that he has just discovered a lump on his breast and is fearful of cancer. The nurse knows that which statement about breast cancer in males is true? a. Breast masses in men are difficult to detect because of minimal breast tissue. b. Breast cancer in men rarely spreads to the lymph nodes. c. One percent of all breast cancer occurs in men. d. Most breast masses in men are diagnosed as gynecomastia.

C One percent of all breast cancer occurs in men. Early spread to axillary lymph nodes occurs due to minimal breast tissue.

During a discussion about breast self-examination with a 30-year-old woman, which of these statements by the nurse is most appropriate? a. "The best time to examine your breasts is during ovulation." b. "Examine your breasts every month on the same day of the month." c. "Examine your breasts shortly after your menstrual period each month." d. "The best time to examine your breasts is immediately before menstruation."

C The best time to conduct breast self-examination is shortly after the menstrual period when the breasts are the smallest and least congested.

In performing an assessment of a woman's axillary lymph system, the nurse should assess which of these nodes? a. Central, axillary, lateral, and sternal nodes b. Pectoral, lateral, anterior, and sternal nodes c. Central, lateral, pectoral, and subscapular nodes d. Lateral, pectoral, axillary, and suprascapular nodes

C The breast has extensive lymphatic drainage. Four groups of axillary nodes are present: (1) central, (2) pectoral (anterior), (3) subscapular (posterior), and (4) lateral.

While examining a 75-year-old woman, the nurse notices that the skin over her right breast is thickened and the hair follicles are exaggerated. This condition is known as: a. dimpling. b. retraction. c. peau d'orange. d. benign breast disease.

C This condition is known as peau d'orange. Lymphatic obstruction produces edema, which thickens the skin and exaggerates the hair follicles. The skin has a pig-skin or orange-peel look, and this condition suggests cancer.

A 55-year-old postmenopausal woman is being seen in the clinic for a yearly examination. She is concerned about changes in her breasts that she has noticed over the past 5 years. She states that her breasts have decreased in size and that the elasticity has changed so that her breasts seem "flat and flabby." The nurse's best reply would be: a. "This change occurs most often because of long-term use of bras that do not provide enough support to the breast tissues." b. "This is a normal change that occurs as women get older. It is due to the increased levels of progesterone during the aging process." c. "Decreases in hormones after menopause causes atrophy of the glandular tissue in the breast. This is a normal process of aging." d. "Postural changes in the spine make it appear that your breasts have changed in shape. Exercises to strengthen the muscles of the upper back and chest wall will help to prevent the changes in elasticity and size."

C The hormonal changes of menopause cause the breast glandular tissue to atrophy, making the breasts more pendulous, flattened, and sagging.

During the physical examination, the nurse notices that a female patient has an inverted left nipple. Which statement regarding this is most accurate? a. Normal nipple inversion is usually bilateral. b. A unilateral inversion of a nipple is always a serious sign. c. It should be determined whether the inversion is a recent change. d. Nipple inversion is not significant unless accompanied by an underlying palpable mass.

C The nurse should distinguish a recently retracted nipple from one that has been inverted for many years or since puberty. Normal nipple inversion may be unilateral or bilateral and usually can be pulled out (i.e., it is not fixed). Recent nipple retraction signifies acquired disease. See Table 17-3.

The nurse is conducting a class about breast self-examination (BSE). Which of these statements indicates proper BSE technique? a. The best time to perform BSE is in the middle of the menstrual cycle. b. The woman needs to do BSE only bimonthly unless she has fibrocystic breast tissue. c. The best time to perform BSE is 4 to 7 days after the first day of the menstrual period. d. If she suspects that she is pregnant, the woman should not perform a BSE until her baby is born.

C The nurse should help each woman establish a regular schedule of self-care. The best time to conduct breast self-examination is right after the menstrual period, or the fourth through seventh day of the menstrual cycle, when the breasts are the smallest and least congested. Advise the pregnant or menopausal woman who is not having menstrual periods to select a familiar date to examine her breasts each month, for example, her birth date or the day the rent is due.

During an examination of a woman, the nurse notices that her left breast is slightly larger than her right breast. Which of these statements is true about this finding? a. Breasts should always be symmetric. b. This finding is probably due to breastfeeding and is nothing to worry about. c. This finding is not unusual, but the nurse should verify that this change is not new. d. This finding is very unusual and means she may have an inflammation or growth.

C The nurse should notice symmetry of size and shape. It is common to have a slight asymmetry in size; often the left breast is slightly larger than the right. A sudden increase in the size of one breast signifies inflammation or new growth.

The nurse is preparing to teach a woman about breast self-examination (BSE). Which statement by the nurse is correct? a. "BSE is more important than ever for you because you have never had any children." b. "BSE is so important because one out of nine women will develop breast cancer in her lifetime." c. "BSE on a monthly basis will help you feel familiar with your own breasts and their normal variations." d. "BSE will save your life because you are likely to find a cancerous lump between mammograms."

C The nurse should stress that a regular monthly self-examination will familiarize her with her own breasts and their normal variations. This is a positive step that will reassure her of her healthy state. While teaching, the nurse should focus on the positive aspects of breast self-examination and should avoid citing frightening mortality statistics about breast cancer. This may generate excessive fear and denial that actually obstructs a woman's self-care action.

During a history interview, a female patient states that she has noticed a few drops of clear discharge from her right nipple. What should the nurse do next? a. Contact the physician immediately to report the discharge. b. Ask her if she is possibly pregnant. c. Ask her some additional questions about the medications she is taking. d. Immediately obtain a sample for culture and sensitivity testing.

C The use of some medications, such as oral contraceptives, phenothiazines, diuretics, digitalis, steroids, methyldopa, and calcium channel blockers, may cause clear nipple discharge. Bloody or blood-tinged discharge from the nipple, not clear, is significant, especially if a lump is also present. In the pregnant female, colostrum would be a thick, yellowish liquid, and it would be expressed after the fourth month of pregnancy.

When a breastfeeding mother is diagnosed with a breast abscess, which of these instructions from the nurse is correct? The mother needs to: a. continue to nurse on both sides to encourage milk flow. b. discontinue nursing immediately to allow for healing. c. temporarily discontinue nursing on affected breast and manually express milk and discard it. d. temporarily discontinue nursing on affected breast but can manually express milk and give it to the baby.

C With a breast abscess, the patient must temporarily discontinue nursing on the affected breast, manually express the milk, and discard it. Nursing can continue on the unaffected side.

7. Generally, the changes normally associated with menopause occur because the cells in the reproductive tract are: a. Aging. b. Becoming fibrous. c. Estrogen dependent. d. Able to respond to estrogen.

C Because cells in the reproductive tract are estrogen dependent, decreased estrogen levels during menopause bring dramatic physical changes. The other options are not correct. DIF: Cognitive Level: Remembering (Knowledge) REF: p. 740 MSC: Client Needs: Health Promotion and Maintenance

11. A patient has had three pregnancies and two live births. The nurse would record this information as grav _____, para _____, AB _____. a. 2; 2; 1 b. 3; 2; 0 c. 3; 2; 1 d. 3; 3; 1

C Gravida (grav) is the number of pregnancies. Para is the number of births. Abortions are interrupted pregnancies, including elective abortions and spontaneous miscarriages. DIF: Cognitive Level: Applying (Application) REF: p. 741 MSC: Client Needs: Safe and Effective Care Environment: Management of Care

43. During an external genitalia examination of a woman, the nurse notices several lesions around the vulva. The lesions are pink, moist, soft, and pointed papules. The patient states that she is not aware of any problems in that area. The nurse recognizes that these lesions may be: a. Syphilitic chancre. b. Herpes simplex virus type 2 (herpes genitalis). c. HPV or genital warts. d. Pediculosis pubis (crab lice).

C HPV lesions are painless, warty growths that the woman may not notice. Lesions are pink or flesh colored, soft, pointed, moist, warty papules that occur in single or multiple cauliflower-like patches around the vulva, introitus, anus, vagina, or cervix. Herpetic lesions are painful clusters of small, shallow vesicles with surrounding erythema. Syphilitic chancres begin as a solitary silvery papule that erodes into a red, round or oval superficial ulcer with a yellowish discharge. Pediculosis pubis causes severe perineal itching and excoriations and erythematous areas (see Table 26-2). DIF: Cognitive Level: Analyzing (Analysis) REF: p. 765 MSC: Client Needs: Physiologic Integrity: Physiologic Adaptation

12. During the interview with a female patient, the nurse gathers data that indicate the patient is perimenopausal. Which of these statements made by this patient leads to this conclusion? a. "I have noticed that my muscles ache at night when I go to bed." b. "I will be very happy when I can stop worrying about having a period." c. "I have been noticing that I sweat a lot more than I used to, especially at night." d. "I have only been pregnant twice, but both times I had breast tenderness as my first symptom."

C Hormone shifts occur during the perimenopausal period, and associated symptoms of menopause may occur, such as hot flashes, night sweats, numbness and tingling, headache, palpitations, drenching sweats, mood swings, vaginal dryness, and itching. The other responses are not correct. DIF: Cognitive Level: Analyzing (Analysis) REF: p. 741 MSC: Client Needs: Health Promotion and Maintenance

29. The nurse is examining a 35-year-old female patient. During the health history, the nurse notices that she has had two term pregnancies, and both babies were delivered vaginally. During the internal examination, the nurse observes that the cervical os is a horizontal slit with some healed lacerations and that the cervix has some nabothian cysts that are small, smooth, and yellow. In addition, the nurse notices that the cervical surface is granular and red, especially around the os. Finally, the nurse notices the presence of stringy, opaque, odorless secretions. Which of these findings are abnormal? a. Nabothian cysts are present. b. The cervical os is a horizontal slit. c. The cervical surface is granular and red. d. Stringy and opaque secretions are present.

C Normal findings: Nabothian cysts may be present on the cervix after childbirth. The cervical os is a horizontal, irregular slit in the parous woman. Secretions vary according to the day of the menstrual cycle, and may be clear and thin or thick, opaque, and stringy. The surface is normally smooth, but cervical eversion, or ectropion, may occur where the endocervical canal is rolled out. Abnormal finding: The cervical surface should not be reddened or granular, which may indicate a lesion. DIF: Cognitive Level: Analyzing (Analysis) REF: p. 750 MSC: Client Needs: Health Promotion and Maintenance

32. When performing the bimanual examination, the nurse notices that the cervix feels smooth and firm, is round, and is fixed in place (does not move). When cervical palpation is performed, the patient complains of some pain. The nurse's interpretation of these results should be which of these? a. These findings are all within normal limits. b. Cervical consistency should be soft and velvety—not firm. c. The cervix should move when palpated; an immobile cervix may indicate malignancy. d. Pain may occur during palpation of the cervix.

C Normally, the cervix feels smooth and firm, similar to the consistency of the tip of the nose. It softens and feels velvety at 5 to 6 weeks of pregnancy (Goodell sign). The cervix should be evenly rounded. With a finger on either side, the examiner should be able to move the cervix gently from side to side, and doing so should produce no pain for the patient. Hardness of the cervix may occur with malignancy. Immobility may occur with malignancy, and pain may occur with inflammation or ectopic pregnancy. DIF: Cognitive Level: Analyzing (Analysis) REF: p. 755 MSC: Client Needs: Health Promotion and Maintenance

21. When the nurse is interviewing a preadolescent girl, which opening question would be least threatening? a. "Do you have any questions about growing up?" b. "What has your mother told you about growing up?" c. "When did you notice that your body was changing?" d. "I remember being very scared when I got my period. How do you think you'll feel?"

C Open-ended questions such as, "When did you ...?" rather than "Do you ...?" should be asked. Open-ended questions are less threatening because they imply that the topic is normal and unexceptional. DIF: Cognitive Level: Understanding (Comprehension) REF: p. 743 MSC: Client Needs: Psychosocial Integrity

41. During a bimanual examination, the nurse detects a solid tumor on the ovary that is heavy and fixed, with a poorly defined mass. This finding is suggestive of: a. Ovarian cyst. b. Endometriosis. c. Ovarian cancer. d. Ectopic pregnancy.

C Ovarian tumors that are solid, heavy, and fixed, with poorly defined mass are suggestive of malignancy. Benign masses may feel mobile and solid. An ovarian cyst may feel smooth, round, fluctuant, mobile, and nontender. With an ectopic pregnancy, the examiner may feel a palpable, tender pelvic mass that is solid, mobile, and unilateral. Endometriosis may have masses (in various locations in the pelvic area) that are small, firm, nodular, and tender to palpation, with enlarged ovaries. DIF: Cognitive Level: Applying (Application) REF: p. 772 MSC: Client Needs: Physiologic Integrity: Physiologic Adaptation

15. During the interview, a patient reveals that she has some vaginal discharge. She is worried that it may be a sexually transmitted infection. The nurse's most appropriate response to this would be: a. "Oh, don't worry. Some cyclic vaginal discharge is normal." b. "Have you been engaging in unprotected sexual intercourse?" c. "I'd like some information about the discharge. What color is it?" d. "Have you had any urinary incontinence associated with the discharge?"

C Questions that help the patient reveal more information about her symptoms should be asked in a nonthreatening manner. Asking about the amount, color, and odor of the vaginal discharge provides the opportunity for further assessment. Normal vaginal discharge is small, clear or cloudy, and always nonirritating. DIF: Cognitive Level: Analyzing (Analysis) REF: p. 742 MSC: Client Needs: Physiologic Integrity: Physiologic Adaptation

16. A woman states that 2 weeks ago she had a urinary tract infection that was treated with an antibiotic. As a part of the interview, the nurse should ask, "Have you noticed any: a. "Changes in your urination patterns?" b. "Excessive vaginal bleeding?" c. "Unusual vaginal discharge or itching?" d. "Changes in your desire for intercourse?"

C Several medications may increase the risk of vaginitis. Broad-spectrum antibiotics alter the balance of normal flora, which may lead to the development of vaginitis. The other questions are not appropriate. DIF: Cognitive Level: Applying (Application) REF: p. 742 MSC: Client Needs: Physiologic Integrity: Pharmacologic and Parenteral Therapies

27. The nurse is preparing for an internal genitalia examination of a woman. Which order of the examination is correct? a. Bimanual, speculum, and rectovaginal b. Speculum, rectovaginal, and bimanual c. Speculum, bimanual, and rectovaginal d. Rectovaginal, bimanual, and speculum

C The correct sequence is speculum examination, then bimanual examination after removing the speculum, and then rectovaginal examination. The examiner should change gloves before performing the rectovaginal examination to avoid spreading any possible infection. DIF: Cognitive Level: Analyzing (Analysis) REF: p. 748 |p. 754 |p. 758 MSC: Client Needs: Safe and Effective Care Environment: Management of Care

28. During an internal examination of a woman's genitalia, the nurse will use which technique for proper insertion of the speculum? a. The woman is instructed to bear down, the speculum blades are opened and applied in a swift, upward movement. b. The blades of the speculum are inserted on a horizontal plane, turning them to a 30-degree angle while continuing to insert them. The woman is asked to bear down after the speculum is inserted. c. The woman is instructed to bear down, the width of the blades are horizontally turned, and the speculum is inserted downward at a 45-degree angle toward the small of the woman's back. d. The blades are locked open by turning the thumbscrew. Once the blades are open, pressure is applied to the introitus and the blades are inserted downward at a 45-degree angle to bring the cervix into view.

C The examiner should instruct the woman to bear down, turn the width of the blades horizontally, and insert the speculum at a 45-degree angle downward toward the small of the woman's back. (See the text under "Speculum Examination" for more detail.) DIF: Cognitive Level: Applying (Application) REF: p. 749 MSC: Client Needs: Safe and Effective Care Environment: Management of Care

17. Which statement would be most appropriate when the nurse is introducing the topic of sexual relationships during an interview? a. "Now, it is time to talk about your sexual history. When did you first have intercourse?" b. "Women often feel dissatisfied with their sexual relationships. Would it be okay to discuss this now?" c. "Women often have questions about their sexual relationship and how it affects their health. Do you have any questions?" d. "Most women your age have had more than one sexual partner. How many would you say you have had?"

C The nurse should begin with an open-ended question to assess individual needs. The nurse should include appropriate questions as a routine part of the health history, because doing so communicates that the nurse accepts the individual's sexual activity and believes it is important. The nurse's comfort with the discussion prompts the patient's interest and, possibly, relief that the topic has been introduced. The initial discussion establishes a database for comparison with any future sexual activities and provides an opportunity to screen sexual problems. DIF: Cognitive Level: Applying (Application) REF: p. 742 MSC: Client Needs: Psychosocial Integrity

25. An 18-year-old patient is having her first pelvic examination. Which action by the nurse is appropriate? a. Inviting her mother to be present during the examination b. Avoiding the lithotomy position for this first time because it can be uncomfortable and embarrassing c. Raising the head of the examination table and giving her a mirror so that she can view the examination d. Fully draping her, leaving the drape between her legs elevated to avoid embarrassing her with eye contact

C The techniques of the educational or mirror pelvic examination should be used. This is a routine examination with some modifications in attitude, position, and communication. First, the woman is considered an active participant, one who is interested in learning and in sharing decisions about her own health care. The woman props herself up on one elbow, or the head of the table is raised. Her other hand holds a mirror between her legs, above the examiner's hands. The young woman can see all that the examiner is doing and has a full view of her genitalia. The mirror works well for teaching normal anatomy and its relationship to sexual behavior. The examiner can ask her if she would like to have a family member, friend, or chaperone present for the examination. The drape should be pushed down between the patient's legs so that the nurse can see her face. DIF: Cognitive Level: Applying (Application) REF: p. 746 MSC: Client Needs: Safe and Effective Care Environment: Management of Care

40. A 46-year-old woman is in the clinic for her annual gynecologic examination. She voices a concern about ovarian cancer because her mother and sister died of it. Which statement does the nurse know to be correct regarding ovarian cancer? a. Ovarian cancer rarely has any symptoms. b. The Pap smear detects the presence of ovarian cancer. c. Women at high risk for ovarian cancer should have annual transvaginal ultrasonography for screening. d. Women over age 40 years should have a thorough pelvic examination every 3 years.

C With ovarian cancer, the patient may have abdominal pain, pelvic pain, increased abdominal size, bloating, and nonspecific gastrointestinal symptoms; or she may be asymptomatic. The Pap smear does not detect the presence of ovarian cancer. Annual transvaginal ultrasonography may detect ovarian cancer at an earlier stage in women who are at high risk for developing it. DIF: Cognitive Level: Applying (Application) REF: p. 772 MSC: Client Needs: Health Promotion and Maintenance

Plugged duct:

Common when milk is not removed completely because of poor latching, ineffective cycling, and frequent nursing, or switching to second breast too soon.

. When providing culturally competent care, nurses must incorporate cultural assessments into their health assessments. Which statement is most appropriate to use when initiating an assessment of cultural beliefs with an older American-Indian patient? a. Are you of the Christian faith? b. Do you want to see a medicine man? c. How often do you seek help from medical providers? d. What cultural or spiritual beliefs are important to you?

D

A 63-year-old Chinese-American man enters the hospital with complaints of chest pain, shortness of breath, and palpitations. Which statement most accurately reflects the nurses best course of action? a. The nurse should focus on performing a full cardiac assessment. b. The nurse should focus on psychosomatic complaints because the patient has just learned that his wife has cancer. c. This patient is not in any danger at present; therefore, the nurse should send him home with instructions to contact his physician. d. It is unclear what is happening with this patient; consequently, the nurse should perform an assessment in both the physical and the psychosocial realms.

D

After a symptom is recognized, the first effort at treatment is often self-care. Which of the following statements about self-care is true? Self-care is: a. Not recognized as valuable by most health care providers. b. Usually ineffective and may delay more effective treatment. c. Always less expensive than biomedical alternatives. d. Influenced by the accessibility of over-the-counter medicines.

D

During a class on religion and spirituality, the nurse is asked to define spirituality. Which answer is correct? Spirituality: a. Is a personal search to discover a supreme being. b. Is an organized system of beliefs concerning the cause, nature, and purpose of the universe. c. Is a belief that each person exists forever in some form, such as a belief in reincarnation or the afterlife. d. Arises out of each persons unique life experience and his or her personal effort to find purpose in life.

D

During a class on the aspects of culture, the nurse shares that culture has four basic characteristics. Which statement correctly reflects one of these characteristics? a. Cultures are static and unchanging, despite changes around them. b. Cultures are never specific, which makes them hard to identify. c. Culture is most clearly reflected in a persons language and behavior. d. Culture adapts to specific environmental factors and available natural resources.

D

In the hot/cold theory, illnesses are believed to be caused by hot or cold entering the body. Which of these patient conditions is most consistent with a cold condition? a. Patient with diabetes and renal failure b. Teenager with an abscessed tooth c. Child with symptoms of itching and a rash d. Older man with gastrointestinal discomfort

D

The nurse is reviewing the hot/cold theory of health and illness. Which statement best describes the basic tenets of this theory? a. The causation of illness is based on supernatural forces that influence the humors of the body. b. Herbs and medicines are classified on their physical characteristics of hot and cold and the humors of the body. c. The four humors of the body consist of blood, yellow bile, spiritual connectedness, and social aspects of the individual. d. The treatment of disease consists of adding or subtracting cold, heat, dryness, or wetness to restore the balance of the humors of the body.

D

The nurse manager is explaining culturally competent care during a staff meeting. Which statement accurately describes the concept of culturally competent care? The caregiver: a. Is able to speak the patients native language. b. Possesses some basic knowledge of the patients cultural background. c. Applies the proper background knowledge of a patients cultural background to provide the best possible health care. d. Understands and attends to the total context of the patients situation.

D

The nurse recognizes that categories such as ethnicity, gender, and religion illustrate the concept of: a. Family. b. Cultures. c. Spirituality. d. Subcultures.

D

When discussing the use of the term subculture, the nurse recognizes that it is best described as: a. Fitting as many people into the majority culture as possible. b. Defining small groups of people who do not want to be identified with the larger culture. c. Singling out groups of people who suffer differential and unequal treatment as a result of cultural variations. d. Identifying fairly large groups of people with shared characteristics that are not common to all members of a culture.

D

A 9-year-old girl is in the clinic for a sports physical. After some initial shyness she finally asks, *"Am I normal? I don't seem to need a bra yet, but I have some friends who do. What if I never get breasts?"* The nurse's best response would be: a. "Don't worry, you still have plenty of time to develop." b. "I know just how you feel, I was a late bloomer myself. Just be patient and they will grow." c. "You will probably get your periods before you notice any significant growth in your breasts." d. "I understand that it is hard to feel different from your friends. Breasts usually develop between 8 and 10 years of age."

D Adolescent breast development usually begins between 8 and 10 years of age. The nurse should not belittle the girl's feelings by using statements like "don't worry" or by sharing personal experiences. The beginning of breast development precedes menarche by about 2 years.

A 65-year-old patient remarks that she just can't believe that her breasts sag so much. She states it must be from lack of exercise. What explanation should the nurse offer her? a. After menopause, only women with large breasts experience sagging. b. After menopause, sagging is usually due to decreased muscle mass within the breast. c. After menopause, a diet that is high in protein will help maintain muscle mass, which keeps the breasts from sagging. d. After menopause, the glandular and fat tissue atrophies, causing breast size and elasticity to diminish, resulting in breasts that sag.

D After menopause, the glandular tissue atrophies and is replaced with connective tissue. The fat envelope atrophies also, beginning in the middle years and becoming marked in the eighth and ninth decades. These changes decrease breast size and elasticity, so the breasts droop and sag, looking flattened and flabby.

The nurse is teaching a pregnant woman about breast milk. Which statement by the nurse is correct? a. "Your breast milk is present immediately after delivery of the baby." b. "Breast milk is rich in protein and sugars (lactose) but has very little fat." c. "The colostrum, which is present right after birth, does not contain the same nutrition as breast milk does." d. "You may notice a thick, yellow fluid expressed from your breasts as early as the fourth month of pregnancy."

D After the fourth month, colostrum may be expressed. This thick yellow fluid is the precursor of milk, and it contains the same amount of protein and lactose but practically no fat. The breasts produce colostrum for the first few days after delivery. It is rich with antibodies that protect the newborn against infection, so breastfeeding is important.

The nurse is performing a breast examination. Which of these statements best describes the correct procedure to use when screening for nipple and skin retraction during a breast examination? Have the woman: a. bend over and touch her toes. b. lie down on her left side and notice any retraction. c. shift from a supine position to a standing position; notice any lag or retraction. d. slowly lift her arms above her head and note any retraction or lag in movement.

D Direct the woman to change position while checking the breasts for skin retraction signs. First ask her to lift her arms slowly over her head. Both breasts should move up symmetrically. Retraction signs are due to fibrosis in the breast tissue, usually caused by growing neoplasms. The nurse should notice if there is a lag in movement of one breast.

Which of these clinical situations would the nurse consider to be outside normal limits? a. A patient has had one pregnancy. She states that she believes she may be entering menopause. Her breast examination reveals breasts that are soft and sag slightly. b. A patient has never been pregnant. Her breast examination reveals large pendulous breasts that have a firm, transverse ridge along the lower quadrant in both breasts. c. A patient has never been pregnant. She reports that she should begin her period tomorrow. Her breast examination reveals breast tissue that is nodular and somewhat engorged. She states that the examination was slightly painful. d. A patient has had two pregnancies and she breastfed both of her children. Her youngest child is now 10 years old. Her breast examination reveals breast tissue that is somewhat soft and she has a small amount of thick yellow discharge from both nipples.

D In nulliparous women, normal breast tissue feels firm, smooth, and elastic; after pregnancy, the tissue feels softer and looser. If any discharge appears, the nurse should note its color and consistency. Except in pregnancy and lactation, discharge is abnormal. Premenstrual engorgement is normal, and consists of a slight enlargement, tenderness to palpation, and a generalized nodularity. A firm, transverse ridge of compressed tissue in the lower quadrants, known as the inframammary ridge, is especially noticeable in large breasts.

47. A 35-year-old woman is at the clinic for a gynecologic examination. During the examination, she asks the nurse, "How often do I need to have this Pap test done?" Which reply by the nurse is correct? a. "It depends. Do you smoke?" b. "A Pap test needs to be performed annually until you are 65 years of age." c. "If you have two consecutive normal Pap tests, then you can wait 5 years between tests." d. "After age 30 years, if you have three consecutive normal Pap tests, then you may be screened every 2 to 3 years."

D Cervical cancer screening with the Pap test continues annually until age 30 years. After age 21, regardless of sexual history or activity, women should be screened every 3 years until age 30, then every 5 years until age 65. DIF: Cognitive Level: Applying (Application) REF: p. 741 MSC: Client Needs: Health Promotion and Maintenance

38. A 22-year-old woman is being seen at the clinic for problems with vulvar pain, dysuria, and fever. On physical examination, the nurse notices clusters of small, shallow vesicles with surrounding erythema on the labia. Inguinal lymphadenopathy present is also present. The most likely cause of these lesions is: a. Pediculosis pubis. b. Contact dermatitis. c. HPV. d. Herpes simplex virus type 2.

D Herpes simplex virus type 2 exhibits clusters of small, shallow vesicles with surrounding erythema that erupt on the genital areas. Inguinal lymphadenopathy is also present. The woman reports local pain, dysuria, and fever. (See Table 26-2 for more information and the descriptions of the other conditions.) DIF: Cognitive Level: Analyzing (Analysis) REF: p. 764 MSC: Client Needs: Physiologic Integrity: Physiologic Adaptation

34. A 65-year-old woman is in the office for routine gynecologic care. She had a complete hysterectomy 3 months ago after cervical cancer was detected. Which statement does the nurse know to be true regarding this visit? a. Her cervical mucosa will be red and dry looking. b. She will not need to have a Pap smear performed. c. The nurse can expect to find that her uterus will be somewhat enlarged and her ovaries small and hard. d. The nurse should plan to lubricate the instruments and the examining hand adequately to avoid a painful examination.

D In the aging adult woman, natural lubrication is decreased; therefore, to avoid a painful examination, the nurse should take care to lubricate the instruments and the examining hand adequately. Menopause, with the resulting decrease in estrogen production, shows numerous physical changes. The cervix shrinks and looks pale and glistening. With the bimanual examination, the uterus feels smaller and firmer and the ovaries are not normally palpable. Women should continue cervical cancer screening up to age 65 years if they have an intact cervix and are in good health. Women who have had a total hysterectomy do not need cervical cancer screening if they have 3 consecutive negative Pap tests or 2 or more consecutive negative HIV and Pap tests within the last 10 years. DIF: Cognitive Level: Applying (Application) REF: p. 760 MSC: Client Needs: Health Promotion and Maintenance

18. A 22-year-old woman has been considering using oral contraceptives. As a part of her health history, the nurse should ask: a. "Do you have a history of heart murmurs?" b. "Will you be in a monogamous relationship?" c. "Have you carefully thought this choice through?" d. "If you smoke, how many cigarettes do you smoke per day?"

D Oral contraceptives, together with cigarette smoking, increase the risk for cardiovascular side effects. If cigarettes are used, then the nurse should assess the patient's smoking history. The other questions are not appropriate. DIF: Cognitive Level: Applying (Application) REF: p. 743 MSC: Client Needs: Physiologic Integrity: Pharmacologic and Parenteral Therapies

6. A woman who is 8 weeks pregnant is in the clinic for a checkup. The nurse reads on her chart that her cervix is softened and looks cyanotic. The nurse knows that the woman is exhibiting __________ sign and __________ sign. a. Tanner; Hegar b. Hegar; Goodell c. Chadwick; Hegar d. Goodell; Chadwick

D Shortly after the first missed menstrual period, the female genitalia show signs of the growing fetus. The cervix softens (Goodell sign) at 4 to 6 weeks, and the vaginal mucosa and cervix look cyanotic (Chadwick sign) at 8 to 12 weeks. These changes occur because of increased vascularity and edema of the cervix and hypertrophy and hyperplasia of the cervical glands. Hegar sign occurs when the isthmus of the uterus softens at 6 to 8 weeks. Tanner sign is not a correct response. DIF: Cognitive Level: Understanding (Comprehension) REF: p. 739 MSC: Client Needs: Health Promotion and Maintenance

1. During a health history, a 22-year old woman asks, "Can I get that vaccine for human papilloma virus (HPV)? I have genital warts and I'd like them to go away!" What is the nurse's best response? a. "The HPV vaccine is for girls and women ages 9 to 26 years, so we can start that today." b. "This vaccine is only for girls who have not yet started to become sexually active." c. "Let's check with the physician to see if you are a candidate for this vaccine." d. "The vaccine cannot protect you if you already have an HPV infection."

D The HPV vaccine is appropriate for girls and women age 9 to 26 years and is administered to prevent cervical cancer by preventing HPV infections before girls become sexually active. However, it cannot protect the woman if an HPV infection is already present. DIF: Cognitive Level: Analyzing (Analysis) REF: p. 740 MSC: Client Needs: General

42. A 25-year-old woman comes to the emergency department with a sudden fever of 38.3° C and abdominal pain. Upon examination, the nurse notices that she has rigid, boardlike lower abdominal musculature. When the nurse tries to perform a vaginal examination, the patient has severe pain when the uterus and cervix are moved. The nurse knows that these signs and symptoms are suggestive of: a. Endometriosis. b. Uterine fibroids. c. Ectopic pregnancy. d. Pelvic inflammatory disease.

D These signs and symptoms are suggestive of acute pelvic inflammatory disease, also known as acute salpingitis (see Table 26-7). (For the descriptions of endometriosis and uterine fibroids, see Table 26-6; for a description of ectopic pregnancy, see Table 26-7.) DIF: Cognitive Level: Analyzing (Analysis) REF: p. 771 MSC: Client Needs: Physiologic Integrity: Physiologic Adaptation

b

should you: a. speculate on what caused an injury b. document what you observed and what the victim said

d

Dehydration and malnutrition can be manifestations of ________________ in older adults. a. physical abuse b. intimate partner violence c. psychological abuse d. neglect

b

During an interview, a woman has answered "yes" to 3 of the Abuse Assessment Screen questions. How should you proceed? a. ask the patient if she has filed a restraining order b. proceed by asking more questions about the items she answered "yes." c. respond by confirming that the patient was abused d. interview the woman's partner and compare notes

d

During examination, you notice a patterned injury on a patient's back. Which is most likely? a. a blunt force trauma b. a friction rub c. a stabbing d. a welt from a leather belt

Paget's disease (intraductal carcinoma)

Early lesion has unilateral, clear, yellow discharge and dry, scaling crusts, friable at nipple apex. Spreads out word to the Ariola with erythematous halo on areola and a crusted retracted nipple. Symptoms include tingling, burning, itching except for the redness and occasional cracking from initial breast-feeding any dermatitis of the nipple area must be explored carefully and referred immediately

e

Elder abuse and neglect assessments include: a. willful infliction of force b. withholding prescription medications without medical orders c. not replacing broken eyeglasses d. threatening to place someone in a nursing home e. all of the above

How is the breast divided when doing an exam?

Four quadrants; upper inner quadrant upper outer quadrant lower inner quadrant Lower outer quadrant axillary tail of Spence

How does being overweight affect development and girls?

Girls Who are overweight or obese BMI levels have a significantly higher occurrence of early breast budding and early menarche

How to teach a self breast exam

Goal is for the woman to become familiar with the look and feel of her breasts that she can detect any change to report promptly. It is best to perform right after menstrual period day for today seven of the cycle when the brass are the smallest and least congested

How do you inspect and palpate the axilla?

Have a woman sitting up you can hold her arm or have her put her arms on her hips. And then we will move your hand to palpate the axilla down the chest wall in a line from the middle of the axilla along the anterior border of axilla along the posterior border and along the anterior aspect of the arm. Move the woman's arm through range of motion to increase the surface area you can reach.

How do you palpate the breast?

Have the woman lay supine position with arms raise above the head this will flatten the breast tissue and this place immediately significant lamps will be more distinct. you also want to make sure that you use your fingers and make gentle rotary motion to the breast tissue. Very your pressure for light medium and deep tissue in each direction. Three common patterns:is nipple palpation out to the peripheral as following spokes on the wheel or palpating and concrete circular out to the periphery or a vertical stripe pattern

a

Increased bruising and bleeding in older adults may be r/t which of the following? a. Ingestion of nonsteroidal anti-inflammatory drugs b. A reduction in the integrity of blood vessels c. Thinning of the skin d. Decreased fluid intake

mastitis

Inflammatory mass before abscess formation. Usually occurs in a single quadrant. Area is red, swollen, tender, very hot, and hard. Symptoms include: headache malaise fever chills sweating increased pulse flu like symptoms. May occur during the first four months of lactation from infection or from status from a plugged duct. Treatment with rest local heater area anabiotic's and frequent nursing to keep breast as empty as possible. Usually resolves and 2 to 3 days

Tanner stage 5 girls

Mature breasts. Only the nipple protrudes, the Ariola is flush with the breast contour

Benign fibrocystic breast disease:

Multiple tender masses occur with numerous symptoms and physical findings: Swelling and tenderness (cycle discomfort) Mastalgia (severe pain, both cycle and non cyclic) Nodularity (significant lumps both cyclic and noncyclic) Dominant lumps (including cyst and fibroadenomas) Nipple discharge (including intraductal papillomas and duct ectasia) Infection and inflammation (including sub Ariola abscess, lactation mastitis, breast abscess, and mondor disease)

b

Patients are at high risk for which mental health problem associated with intimate partner violence? a. Hallucinations b. Suicidality c. Schizophrenia d. Attention-deficit/hyperactivity disorder (ADHD)

How does being pregnant affect the breast?

Pregnancy stimulates the expansion of the ductile system and supporting fatty tissues and development of the true secretory alveoli. This causes breast to enlarge and feel more knowledgeable. The nipples grow larger, darker and more erect

a

Risk factors for intimate partner homicide include: a. Abuse during pregnancy b. Victim substance abuse c. Victim unemployment d. History of victim childhood sexual assault

a

Routine universal screening for domestic violence includes: a. Asking everyone each time they come to the health care system if they are abused b. Asking everyone who has injuries if they are abused c. Asking everyone who has symptoms of depression and PTSD if they are abused d. Asking everyone ages 18 to 30 years if they are abused

Dimpling in the breast is?

Shallow dimple (also called the skin tether) are shown here is a sign of skin retraction. Cancer causes fibrosis, Which contracts this and spent three ligaments. The dimple may be a parent at rest, with compression or with lifting of the arm. Also note the distortion of the Ariola here as the nipple pulled toward it

Having the tumor suppressor gene BRCA1 or BRCA2 does what?

Significantly increases the risk of developing breast or ovarian cancer

Breast cancer

Solitary, unilateral, nontender mass. Single focus in one area, although it may be enter spread with other nodules. It is hard solid dense and fixes to the underlying tissue or skin as cancer becomes invasive. Borders are irregular and poorly delineated. Constantly growing. Often painless although the person may have pain. Most common in the upper outer quadrant.

*Menarche* refers to

The beginning of menstruation

d

The health care system may help abused women by a. providing financial and supportive services. b. estimating the ages of bruises. c. providing shelter from the abusive individual. d. identifying abuse in the early stages.

How do you differentiate breast lumps?

The likely age, the shape, the consistency, the demarcation, the number, the mobility, if the skin is retracting, the growth pattern, and the risk of health

d

The nurse caring for an older adult suspects older adult abuse. Which action is appropriate? a. Confront the caregivers about the suspicion of abuse. b. Collect proof of abuse before notifying the authorities. c. Report the abuse if the older adult gives permission. d. Notify the authorities of the suspected older adult abuse.

b, d, e

The nurse is assessing a person who is a suspected victim of abuse. When documenting assessment data, which of the following is the most important concept for the nurse to remember? (Select all that apply.) a. Words used when documenting should be sanitized. b. It must be detailed and unbiased. c. Speculation on the cause of injury should be included. d. Use the exact terms the abused person uses to describe sexual organs. e. Quotation marks should be used for severe threats of harm.

d

What is a known risk factor for child maltreatment? a. substance abuse b. intimate partner violence c. physical disability and/or mental retardation in the child d. all of the above

d

When assessing an injury on a child, which should be considered? a. the child's developmental level b. the child's medical and medication history c. the history of how the injury occured d. all of the above

Breast Abscess

Where complication of a generalized infection. Example mastitis if untreated. A pocket of pus that feels hard, looks red, and is quite tender accumulates and one local area. Treated with anabiotic's surgical incision and draining

d

Which clinical situation would require the examiner to report to the proper authorities? a. statements from the victim b. statements from witnesses c. proof of abuse/or neglect d. suspicion of elder abuse and/or neglect

b

Which gynecologic problem is not usually associated with intimate partner violence? a. Pelvic pain b. Ovarian cysts c. STIs d. Vaginal tearing

c

Which is a forensic term that is related to "purpura" but is not related to blunt force trauma? a. wound b. incision c. ecchymosis d. bruise

d

Which scenario is an example of intimate partner violence? a. An ex-boyfriend stalks his ex-girlfriend b. Marital rape c. Hitting a date d. All of the above

b

Which should be routinely included in evaluating a case of elder abuse? a. corroborative interview from caregiver b. baseline laboratory tests c. testing for STIs d. TB testing

a

Which would you include on the chart of the patient who has been the victim of abuse? a. photographic documentation of injuries b. a summary of the abused patient's statements c. verbatim documentation of every statement made d. a general description of injuries in the progress notes

What are the American Cancer Society's recommendations for breast health?

Women 20 to 39 years of age have a clinical breast exam every three years women 40 years and older having an annual mammogram and an annual clinical breast examination conducted close to the same time

b

Women and men who have been physically abused are most often abused by a. a man with a substance use problem. b. an intimate male partner. c. a known sex offender. d. a man convicted of a serious crime.

How is the best way to chart a finding of a lump or mass on a breast exam?

You want to put the location, size, shape, consistency, movable, distinctness, nipple, note the skin over the lump, tenderness and lymphadenopathy

1. A woman has come to the clinic to seek help with a substance abuse problem. She admits to using cocaine just before arriving. Which of these assessment findings would the nurse expect to find when examining this woman? a. Dilated pupils, pacing, and psychomotor agitation b. Dilated pupils, unsteady gait, and aggressiveness c. Pupil constriction, lethargy, apathy, and dysphoria d. Constricted pupils, euphoria, and decreased temperature

a

2. The nurse is assessing a patient who has been admitted for cirrhosis of the liver, secondary to chronic alcohol use. During the physical assessment, the nurse looks for cardiac problems that are associated with chronic use of alcohol, such as: a. Hypertension. b. Ventricular fibrillation. c. Bradycardia. d. Mitral valve prolapse.

a

2. A patient visits the clinic to ask about smoking cessation. He has smoked heavily for 30 years and wants to stop cold turkey. He asks the nurse, What symptoms can I expect if I do this? Which of these symptoms should the nurse share with the patient as possible symptoms of nicotine withdrawal? Select all that apply. a. Headaches b. Hunger c. Sleepiness d. Restlessness e. Nervousness f. Sweating

a, b, d, e

10. A patient is brought to the emergency department. He is restless, has dilated pupils, is sweating, has a runny nose and tearing eyes, and complains of muscle and joint pains. His girlfriend thinks he has influenza, but she became concerned when his temperature went up to 39.4 C. She admits that he has been a heavy drug user, but he has been trying to stop on his own. The nurse suspects that the patient is experiencing withdrawal symptoms from which substance? a. Alcohol b. Heroin c. Crack cocaine d. Sedatives

b

11. The nurse is reviewing aspects of substance abuse in preparation for a seminar. Which of these statements illustrates the concept of tolerance to an illicit substance? The person: a. Has a physiologic dependence on a substance. b. Requires an increased amount of the substance to produce the same effect. c. Requires daily use of the substance to function and is unable to stop using it. d. Experiences a syndrome of physiologic symptoms if the substance is not used.

b

3. The nurse is conducting a class on alcohol and the effects of alcohol on the body. How many standard drinks (each containing 14 grams of alcohol) per day in men are associated with increased deaths from cirrhosis, cancers of the mouth, esophagus, and injuries? a. 2 b. 4 c. 6 d. 8

b

6. When reviewing the use of alcohol by older adults, the nurse notes that older adults have several characteristics that can increase the risk of alcohol use. Which would increase the bioavailability of alcohol in the blood for longer periods in the older adult? a. Increased muscle mass b. Decreased liver and kidney functioning c. Decreased blood pressure d. Increased cardiac output

b

9. The nurse has completed an assessment on a patient who came to the clinic for a leg injury. As a result of the assessment, the nurse has determined that the patient has at-risk alcohol use. Which action by the nurse is most appropriate at this time? a. Record the results of the assessment, and notify the physician on call. b. State, You are drinking more than is medically safe. I strongly recommend that you quit drinking, and Im willing to help you. c. State, It appears that you may have a drinking problem. Here is the telephone number of our local Alcoholics Anonymous chapter. d. Give the patient information about a local rehabilitation clinic.

b

1. A patient with a known history of heavy alcohol use has been admitted to the ICU after he was found unconscious outside a bar. The nurse closely monitors him for symptoms of withdrawal. Which of these symptoms may occur during this time? Select all that apply. a. Bradycardia b. Coarse tremor of the hands c. Transient hallucinations d. Somnolence e. Sweating

b,c, e

4. During a session on substance abuse, the nurse is reviewing statistics with the class. For persons aged 12 years and older, which illicit substance was most commonly used? a. Crack cocaine b. Heroin c. Marijuana d. Hallucinogens

c

8. The nurse is asking an adolescent about illicit substance abuse. The adolescent answers, Yes, Ive used marijuana at parties with my friends. What is the next question the nurse should ask? a. Who are these friends? b. Do your parents know about this? c. When was the last time you used marijuana? d. Is this a regular habit?

c

5. A woman who has just discovered that she is pregnant is in the clinic for her first obstetric visit. She asks the nurse, How many drinks a day is safe for my baby? The nurses best response is: a. You should limit your drinking to once or twice a week. b. Its okay to have up to two glasses of wine a day. c. As long as you avoid getting drunk, you should be safe. d. No amount of alcohol has been determined to be safe during pregnancy.

d

7. During an assessment, the nurse asks a female patient, How many alcoholic drinks do you have a week? Which answer by the patient would indicate at-risk drinking? a. I may have one or two drinks a week. b. I usually have three or four drinks a week. c. Ill have a glass or two of wine every now and then. d. I have seven or eight drinks a week, but I never get drunk.

d

supernumerary nipple

minute extra nipple along the embryonic milk line

Gynecomastia is:

overdevelopment of breast tissue in males

Tanner stage 1 girls

preadolescent no pubic hair

mammary duct ectasia

subacute inflammation of ductal system causes dilated mammary ducts. It's a paste like matter and subAreolar ducts pretty sticky, purulent discharge that maybe white, gray, brown, green or bloody. This itches, burns or drive Payne around the nipple. May have some redness and swelling. Ducks are palpable as Robbie, twisted to bills under the Ariola. May have a palpable mass soft or firm that's poorly delineated not malignant but needs a needle biopsy.


Conjuntos de estudio relacionados

ACS - Alkanes, Alkenes, Alkynes and their Alicyclic Couterparts

View Set

Procedural Programming midterm review

View Set

applied mathematics - unit 4: probability in the social science

View Set